Summary NCC Maternal Newborn Nursing (RNC-MNN) Practice Tests & Test Prep Answered 2022-2023.

According to the March of Dimes, there may be four main routes to spontaneous premature labor. They include all except which of the following?

a. infections or inflammation
b. maternal or fetal stress
c. diabetes
d. bleeding
c. diabetes

Diabetes is not a route considered. The fourth route is uterine stretching. Overdistention of the uterus due to multiple gestation, or placental abnormalities can lead to the release of chemicals that stimulate uterine contractions.

The presence of 10^5 or more bacterial colonies per milliliter of urine on two consecutive, clean catch, midstream voided specimens is an indication of:

a. pyelonephritis
b. cystitis
c. thrombophlebitis
d. bacteriuria
d. bacteriuria

Bacteriuria is defined as the presence of 10^5 or more bacterial colonies per milliliter of urine on two consecutive, clean catch, midstream voided specimens. Risk is associated with sickle cell trait, lower socio-economic status, increased parity, and reduced availability of medical care.

Which of the following is least likely to be a risk factor for postpartum hemorrhage?

a. prolonged labor
b. induced or augmented labor
c. prior vaginal birth
d. difficult third stage of labor
c. prior vaginal birth

The least likely to be a risk factor for postpartum hemorrhage is prior vaginal birth. Prior cesarean birth is a risk factor, however. Other risk factors include: multiparity, coagulopathies, and uterine rupture (among others).

At what point postpartum does the infarcted superficial tissue over the placental site slough off?

a. immediately after the placenta is delivered
b. 24 hours after delivery
c. 72 hours after delivery
d. 7-14 postpartum
d. 7-14 postpartum

At this time, the woman may notice an episode of increased vaginal bleeding, which is usually self-limited. Bleeding lasting more than 1 to 2 hours should be evaluated for late postpartum hemorrhage.

During the first few days after birth, which of the following secretions causes strong uterine contractions and a further reduction in size of the uterus?

a. progesterone
b. estrogen
c. prolactin
d. oxytocin
d. oxytocin

During the first few days after birth, oxytocin secretion causes uterine contractions, especially after breastfeeding and in multiparas. Multiparity, multiple gestation, polyhydramnios, and bladder distention can influence uterine size and the progression of uterine involution.

The National Screening Committee timeline states that all babies should be formally reviewed at the latest by what age?

a. 8 weeks
b. 6 weeks
c. 1 week
d. 2 weeks
a. 8 weeks

The developmental assessment of the baby at this point gives the practitioner an opportunity to ensure that the baby’s neurological development is progressing as would be expected.

Transient tachypnea of the newborn is sometimes referred to as which of the following?

a. wet lung syndrome
b. retained lung fluid
c. RDS type II
d. all of the above
d. all of the above

All of these choices are other names for transient tachypnea of the newborn (TTN). The pathophysiology of TTN is excess fluid in the lungs, failure to clear normal fetal lung fluid, or both.

Respiratory distress syndrome (RDS) is the major cause of respiratory distress in the newborn. It accounts for approximately what percentage of all premature infant deaths?

a. 50-70%
b. 20-30%
c. 25-45%
d. 10-20%
a. 50-70%

RDS, also known as hyaline membrane disease, accounts for approximately 50-70% of all premature infant deaths. RDS occurs in 60% of babies born at less than 28 to 34 weeks, and in less than 5% of those born after approximately 34 weeks.

Which of the following would NOT be an appropriate intervention for a preterm newborn?

a. monitor vital signs every 2-4 hours
b. avoid touching the infant as much as possible
c. monitor weight daily
d. monitor intake and output along with electrolyte balance
b. avoid touching the infant as much as possible

This is not an appropriate intervention. The newborn should be provided with appropriate stimulation, such as touch and cuddling.

Which of the following would indicate implied consent to a procedure?

a. the patient voluntarily submits to the procedure
b. the patient consents in writing
c. the patient orally consents
d. the patient consents orally and in writing
a. the patient voluntarily submits to the procedure

When a patient voluntarily submits to the procedure, this indicates implied consent. Consenting in writing or orally is considered expressed consent.

Which of the following is a risk factor for a retained placenta?

a. previous uterine curettage
b. preeclampsia
c. nulliparity
d. all of the above
d. all of the above

Risk factors for a retained placenta include all of the first three choices along with previous retained placenta, augmented labor, birth in a dorsal position, and use of ergotamine. The primary risk associated with retained placenta is hemorrhage.

Immediately postpartum, the lochia consists primarily of which of the following?

a. exudate
b. leukocytes
c. mucus
d. blood
d. blood

Initially, the lochia consists primarily of blood. As a result, it is red or brownish red in color. As the uterine bleeding decreases at approximately 3-5 days postpartum, leukocyte infiltration occurs and the placental implantation site undergoes exfoliation; at this point, the discharge contains some blood but is primarily exudate and leukocytes, and is pinkish brown in color.

An increase in the volume of amniotic fluid is referred to as

a. hydramnios
b. ketoacidosis
c. dystocia
d. retinopathy
a. hydramnios

Hydramnios, or an increase in the volume of amniotic fluid, occurs in 10-20% of pregnant women with diabetes. It is thought to be a result of excessive fetal urination because of fetal hyperglycemia. Preeclampsia occurs more often in diabetic pregnancies, especially when diabetes related vascular changes already exist.

Which of the following would be indicative of false labor?

a. sedation decreases or stops contractions
b. progressive frequency and intensity of contractions
c. discomfort begins in the back, radiating into the abdomen
d. activity increases contractions
a. sedation decreases or stops contractions

If sedation decreases or stops contractions it is an indication of false labor. Other indications include: longer intervals between contractions, discomfort in lower abdomen and groin, and no appreciable change in the cervix.

Cigarette smoking has been linked to which of the following?

a. low birth weight and prematurity
b. gestational diabetes
c. maternal infections
d. fetal infections
a. low birth weight and prematurity

Cigarette smoking has been linked to an increased incidence of low birth weight and prematurity. From a preventative perspective, it is not enough to discourage smoking in pregnant women. The focus must be on discouraging smoking in any woman of childbearing age who may potentially become pregnant.

A shortage of oxygen can have devastating effects on your baby’s growth and development. On average, smoking during pregnancy doubles the chances that

a. the baby will be born late or weight more than 9.5 lb at birth
b. the baby will be born with tremors
c. the baby will be born too early and have a higher chance of obesity
d. the baby will be born too early or weigh less than 5.5 lb at birth
d. the baby will be born too early or weigh less than 5.5 lbs at birth

Smoking also more than doubles the risk of stillbirth.

The recommended weight gain for an underweight woman who becomes pregnant is which of the following?

a. 15-20 lb
b. 28-40 lb
c. 11-20 lb
d. 20-28 lb
b. 28-40 lb

A woman who is underweight (BMI < 18.5) should gain between 28 to 40 pounds during pregnancy. For a woman of normal weight, 25 to 35 pounds is the recommended weight gain.

Cultural frameworks and cultural assessment tools have been developed to guide perinatal nursing practice, but no matter which model is used it should focus on which of the following?

a. kinship, political factors, economic factors, and educational factors
b. person, processes, environment, and outcomes
c. technological factors, religious factors, philosophical factors, and care expressions
d. extended family, stoicism, healthcare decision making, and time orientation
b. person, processes, environment, and outcomes

What constitutes a positive and satisfying birth experience varies from one culture to another. Giving birth is a significant life event, a reflection of a woman’s personal values about childbearing and child rearing, and the expression and symbolic actualization of the union of the parents.

Which of the following is least likely to be a side effect of magnesium sulfate used in preterm labor?

a. hypertension in the mother
b. decreased fetal heart rate variability
c. decreased neonatal tone
d. flushing and nausea in the mother
a. hypertension in the mother

HYPOtension in the mother is a side effect. Other side effects for the mother include: flushing, nausea, blurred vision, headache, lethargy, muscle weakness, and rarely, pulmonary edema and respiratory or cardiac arrest.

What is the typical iron requirement for a pregnant woman with iron deficiency?

a. 60-100 mg/d
b. 30-40 mg/d
c. 25-30 mg/d
d. 150 mg/d
a. 60-100 mg/d

If there is an iron deficiency or a multiple gestation pregnancy, the recommended amount would be 60-100 mg/d. With iron deficiency anemia, 200 mg/d would be required. In the absence of an iron deficiency, 30 mg/d is the recommended amount of iron for a pregnant woman.

Which of the following findings in a postpartum woman in the early postpartum period would NOT be a normal finding?

a. nipples sore but not painful
b. voiding spontaneously
c. hyperreflexia
d. abdominal
c. hyperreflexia

Hyperreflexia is not a normal finding in the early postpartum period. The expected neurological findings include: being oriented to person, place, and time; having bilateral and frontal headaches for the first week; and normal reflexes.

Which of the following hormonal contraceptive methods is a long-acting reversible contraceptive?

a. transdermal patch
b. contraceptive ring
c. subdermal implants
d. contraceptive injections
c. subdermal implants

Subdermal implants are long-acting reversible contraceptives (LARCs), along with IUDs. Because of their lack of dependency upon user actions, LARCs are more effective than SARCs.

The cervix and lower uterine segment immediately postpartum are

a. firm and thin
b. firm and thick
c. thin and flaccid
d. thick and flaccid
c. thin and flaccid

At 2 to 3 days the cervix resumes it’s customary appearance but remains dilated 2 to 3 cm. By the end of the first week, the cervical opening narrows to a diameter of 1 cm.

At 2 to 3 days post-delivery the cervix of a woman is dilated approximately

a. 1 cm
b. 2-3 cm
c. 3-5 cm
d. 6 cm
b. 2-3 cm

At 2 to 3 days post-delivery, the cervix is dilated approximately 2-3 cm. The cervix is soft and floppy after delivery. It narrows to less than 1 cm in diameter by the end of the first week.

The decrease is gastrointestinal muscle tone and motility and the relaxation of abdominal muscles postpartum can result in which of the following?

a. diarrhea
b. nausea
c. gaseous distention
d. hemorrhoids
c. gaseous distention

The decrease in gastrointestinal muscle tone and motility and the relaxation of abdominal muscles postpartum can result in gaseous distention. This can occur during the first 2 to 3 days postpartum.

An increase in hematocrit is seen between day 3 and day 7 post-delivery due to which of the following?

a. hemodilution
b. cessation of blood flowing through the placenta
c. high prolactin levels
d. plasma volume decrease being greater than the loss of red blood cells after birth
d. plasma volume decrease being greater than the loss of red blood cells after birth

The hematocrit returns to pre-pregnancy value in 4 to 8 weeks.

A boggy fundus postpartum may be a sign of uterine atony, which places the patient at risk for postpartum hemorrhage and other complications. In this situation, the nurse should do which of the following?

a. consult with the woman’s physician
b. apply warm pads
c. perform a uterine massage
d. none of the above
c. perform a uterine massage

Uterine massage promotes blood movement out of the uterus. The nurse should also encourage the patient to void, as a full or distended bladder can impede uterine involution and contractions. The nurse is often in the position as the first member of the healthcare team to learn of these warning signs and therefore must take swift action if an issue is suspected.

The postpartum nurse receives a postpartum patient that has had a normal delivery with no complications. A typical policy for taking vital signs post-delivery would be considered

a. every hour x 4
b. every 15 minutes x 1 hour and then every 30 minutes x 2 hours
c. every 5 minutes for 30 minutes and then every 4 hours x 24 hours
d. every 30 minutes x 3 hours
b. every 15 minutes x 1 hour and then every 30 minutes x 2 hours

Every facility will have a policy and procedure for the vital signs of the postpartum patient. The nurse should be familiar with these.

Which of the following would be considered a danger sign in a postpartum woman?

a. temperature higher than 99.5
b. increased urinary output
c. excessive vaginal bleeding
d. lochia turns brown
c. excessive vaginal bleeding

Other danger signs include: a temperature of 100.4 or higher, decreased urinary output less than 30mL/hour, and resumption of bright red blood after lochia has already tuned brown.

The Newborns and Mother’s Health Act of 1996 recommends which of the following as the minimum hospital stay for a cesarean birth?

a. 96 hours
b. 72 hours
c. 48 hours
d. 36 hours
a. 96 hours

The minimum hospital stay for a cesarean birth is 96 hours and 48 hours for a vaginal birth.

Which of the following is an effective way to reduce thromboembolic disorders postpartum?

a. use of analgesics
b. moist compresses
c. prophylactic antibiotics
d. early ambulation
d. early ambulation

Early ambulation after delivery has decreased the incidence of thromboembolic disorders. DVTs are more commonly diagnosed in the antepartum period, whereas pulmonary emboli are more frequently diagnosed in the postpartum period, more often with cesarean delivery.

Mastitis is characterized by all but which of the following?

a. cool to touch
b. fullness
c. pain
d. hardness
a. cool to touch

Mastitis is an infection of the breast surrounding the ducts that is characterized by fullness, pain, warmth (not coolness), and hardness of the breast. It is crucial to differentiate infection from engorgement. Mastitis needs to be treated with antibiotics and the patient is usually encouraged to continue breastfeeding.

Which of the following is least likely to be a risk factor for postpartum depression?

a. prenatal anxiety
b. older maternal age
c. low socioeconomic status
d. unplanned pregnancy
b. older maternal age

A young maternal age rather than older maternal age is more likely to be a risk factor for postpartum depression. Other risk factors include: prenatal depression, low self-esteem, stress of child care, and life stress in general.

Your teaching to a postpartum woman who gave birth yesterday would include all but which of the following?

a. instructions in feeding technique
b. providing the opportunity for the woman to bathe the baby
c. instruction that chills may be experienced for a few days but that they are of no importance
d. instructions to plan at least one rest period per day
c. instruction that chills may be experienced for a few days but that they are of no importance

You would instead instruct the new mother to report any signs of chills, fever, increased lochia, or depressed feeling to her healthcare provider. You might also demonstrate newborn care skills as necessary; and instruct the client to avoid heavy lifting for at least 3 weeks.

Which of the following is least likely to be a risk factor for postpartum hemorrhage?

a. slow labor
b. augmented labor
c. preeclampsia
d. overdistended uterus
a. slow labor

Rapid labor rather than slow labor is a risk factor for postpartum hemorrhage. Other risk factors include a history of postpartum hemorrhage, episiotomy (especially mediolateral), chorioamnionitis, and operative delivery.

At birth, the posterior fontanel of the infant measures

a. 4-6 cm
b. 2-4 cm
c. 1-2 cm
d. less than 0.5 cm
d. less than 0.5 cm

Fontanels are often referred to as soft spots. They are the wide space at the intersections of the sutures. The posterior fontanel is less than 0.5 cm at birth. The anterior fontanel is 4-6 cm at birth.

An incubator or isolette warms the infant by which of the following?

a. radiation
b. conduction combined with radiation
c. convection
d. convection combined with radiation
c. convection

An incubator or isolette warms the infant by convection. A radiant warmer is an open bed with radiant heat panels placed above the infant so that convective and evaporative heat losses are increased. Once an infant’s temperature is normalized, the infant is placed in an open crib with hats and blankets providing thermal support.

The newborn’s general body response to sudden stimulus that is a combination of full extension and abduction of limbs is which of the following reflexes?

a. rooting
b. Babinski reflex
c. Moro reflex
d. palmar reflex
c. Moro reflex

The Moro reflex is also called the startle reflex. It is the newborn’s general body response to sudden stimulus that is a combination of full extension and abduction of limbs. The Moro reflex is one of many reflexes that are seen at birth. It normally disappears after 3 or 4 months.

Which of the following statements about mastitis in postpartum women is least accurate?

a. mastitis usually occurs approximately 2-3 weeks after birth
b. the infection involves the interlobular connective tissue
c. the infection usually involves only one breast
d. mastitis may occur as early as the first postpartum day
d. mastitis may occur as early as the first postpartum day

Mastitis usually occurs approximately 2-3 weeks after birth and may occur as early as the seventh postpartum day. Predisposing factors include milk stasis (from a blocked duct), nipple trauma (cracked or fissured nipples), and poor breastfeeding technique.

The cardinal symptoms of a postpartum infection are

a. elevated temperature, tachycardia, and pain
b. elevated temperature, bradycardia, and pain
c. elevated temperature, hypotension, and skin pallor
d. elevated temperature, tachycardia, and skin pallor
a. elevated temperature, tachycardia, and pain

The nursery should be notified of these findings, although the newborn need not be separated from the mother.

The newborn’s chest is cylindrical. Measured at the nipple line, its circumference is approximately

a. 25 cm
b. 22 cm
c. 38 cm
d. 33 cm
d. 33 cm

This is 2-3 cm less than the infant’s head.

Which of the following statements about thermoregulation in terms of a newborn is least accurate?

a. the newborn’s ability to maintain temperature control after birth is determined by external environmental factors and internal physiologic processes
b. characteristics of newborns that predispose them to heat loss include a large body surface area in relation to body mass and a limited amount of subcutaneous fat
c. peripheral vasoconstriction increases heat loss to the skin surface
d. evaporation is a mechanism of heat loss
c. peripheral vasoconstriction increases heat loss to the skin surface

Peripheral vasoconstriction decreases (not increases) heat loss to the skin surface. Newborns attempt to regulate body temperature through thermogenesis, increased metabolic rate, and increased muscle activity.

A benign, self-limited, asymptomatic skin condition that only occurs during the neonatal period characterized by erythematous papules, vesicles, and occasionally, pustules is which of the following

a. petechiae
b. hemangiomas
c. erythema toxicum
d. cafe au lait spots
c. erythema toxicum

The eruption is characterized by small, erythematous papules, vesicles, and occasionally, pustules. The lesions are usually surrounded by a distinctive diffuse, blotchy, erythematous halo. Individual lesions are transitory, often disappearing within hours and then appearing elsewhere on the body.

Which of the following is NOT an established risk factor for pelvic floor neuropathy?

a. multiparity
b. low birth weight
c. forceps delivery
d. third-degree laceration at delivery
b. low birth weight

Established risk factors for pelvic floor neuropathy includes multiparity, high birth weight, forceps delivery, prolonged active second stage, and third-degree laceration. Most women will recover function within a few months postpartum.

Antimicrobial drugs work in one of two ways – those that inhibit bacterial growth are

a. synergistic
b. antagonistic
c. bactericidal
d. bacteriostatic
d. bacteriostatic

Antimicrobial medications vary with respect to the range of microorganisms they kill or inhibit. Narrow-spectrum antimicrobial kill only limited range, and broad-spectrum kill a wide range of microorganisms.

There are certain care procedures which can decrease the probability of high bilirubin levels in the newborn. Maintaining the newborn’s skin temperature at 97.8 F will avoid cold stress, which can result in which of the following?

a. acidosis
b. breast milk jaundice
c. gastrointestinal dysfunction
d. anemia
a. acidosis

Acidosis decreases available serum albumin-binding sites, weakens albumin-binding powers, and causes elevated unconjugated bilirubin levels.

Neonatal red blood cells have a lifespan of

a. 10 to 20 days
b. 25 to 50 days
c. 50 to 70 days
d. 80 to 100 days
d. 80 to 100 days

This is approximately two-thirds of an adult’s RBC lifespan. Erythropoiesis resumes normally when levels of erythropoietin rise in response to low hemoglobin levels and tissue oxygen needs.

Macrosomia is a potential complication for mothers who have which of the following conditions?

a. hypotension
b. gestational diabetes
c. cardiac disease
d. thyroid disease
b. gestational diabetes

Macrosomia, or birth weight above 4000g is a possible fetal complication for women with gestational diabetes. Macrosomia may cause traumatic vaginal birth, such as with shoulder dystocia, and subsequent birth trauma. IUGR is possible when the mother also has vascular disease.

Which of the following is not one of the three leading causes of maternal death?

a. pulmonary embolism
b. hemorrhage
c. myocardial infarction
d. gestational hypertension
c. myocardial infarction

Pulmonary embolism, hemorrhage, and gestational hypertension are the three leading causes of maternal death. PE can be a complication of DVT. It occurs when fragments of a blood clot dislodge and are carried to the pulmonary artery or one of its branches.

A marked danger of aspiration in the preterm newborn exists due to all but which of the following?

a. poorly developed gag reflex
b. poor sucking reflex
c. poor swallowing reflex
d. small stomach capacity
d. small stomach capacity

A small stomach capacity is not a reason for aspiration in the preterm newborn. Difficulty in meeting high caloric and fluid needs for growth is due to small stomach capacity. An incompetent esophageal cardiac sphincter is also a factor in the danger of aspiration in the preterm newborn.

A blood loss of 1500-2000 mL from postpartum hemorrhage (PPH) would be considered which of the following?

a. mild
b. moderate
c. severe
d. compensated
b. moderate

Moderate postpartum hemorrhage also includes HR > 120 bpm, systolic blood pressure of 70-80 mmHg, tachypnea, urine output of 5-20 mL/hr, restlessness, pallor, and confusion.

You are measuring the heart rate of a newborn female. When you measure her resting heart rate, the average beats per minute should fall into which of the following ranges?

a. 60-110
b. 80-140
c. 120-180
d. 60-115
c. 120-180

Which of the following is NOT a single-gene disorder?

a. Tay-Sachs disease
b. cystic fibrosis
c. Trisomy 18
d. hemophilia A
c. Trisomy 18

Trisomy 18, or Edwards syndrome, results from a chromosome abnormality. The other three choice are single-gene (Mendelian) disorders.

Whitish-yellow cysts containing epithelial cells may be present on the hard palate at birth. These are called

a. nevus simplex
b. Epstein’s pearls
c. milia
d. thrush
b. Epstein’s pearls

Epstein’s pearls disappear within a few weeks.

Which of the following statements about ABO incompatibility is accurate?

a. ABO incompatibility is uncommon in preterm infants
b. ABO incompatibility becomes more severe in further pregnancies
c. ABO incompatibility cannot occur in first-born babies
d. Significant problems with ABO incompatibility occur mostly with babies whose mothers have O blood type and the baby has either A or B blood type
d. Significant problems with ABO incompatibility occur mostly with babies whose mothers have O blood type and the baby has either A or B blood type

ABO incompatibility is a common and generally mild type of hemolytic disease in babies. Premature babies are much more likely to experience severe problems with ABO incompatibility. Unlike hemolytic disease that can result in subsequent babies when a mother has a negative blood group, ABO incompatibility can occur in first-born babies and does not become more severe in further pregnancies.

Which of the following is least likely to be a risk factor for postpartum hemorrhage?

a. augmented labor
b. slow labor
c. preeclampsia
d. Asian ethnicity
b. slow labor

Rapid labor rather than slow labor is a risk factor for postpartum hemorrhage. Other risk factors include a history of postpartum hemorrhage, episiotomy (especially mediolateral), overdistended uterus, chorioamnionitis, and operative delivery.

Anatomical closure of the foramen ovale occurs by

a. one hour
b. one day
c. one month
d. one year
d. one year

Following increased lung perfusion at birth, the pressures in the two atria equalize causing the foramen ovale valve to close. This closure is functional and a patent foramen ovale may persist until pressures stabilize over the next 24-48 hours. Anatomical closure occurs by one year.

Neutral thermal environment (NTE) is the temperature range in which normal body temperature can be maintained with minimal metabolic demands and oxygen consumption. One of the ways to achieve NTE is with a single-walled plastic crib that warms the infant by convection. This is which of the following?

a. radiant warmer
b. incubator
c. open warmer
d. open crib
b. incubator

Persistent bradycardia in a newborn may be the result of all of the following pathologies EXCEPT

a. congenital heart block
b. sepsis
c. congestive heart failure
d. increased intracranial pressure
c. congestive heart failure

CHF is a pathology for tachycardia rather than bradycardia. Bradycardia in a newborn is considered a heart rate of less than 100 beats per minute. Other pathologies include hypoxemia and asphyxia.

Sole creases can help determine gestational age. However, skin dries to a point where sole creases are no longer a valid indicator of gestational age. This occurs at what time after birth?

a. by 12 hours after birth
b. by 8 hours after birth
c. by 6 hours after birth
d. by 2 hours after birth
a. by 12 hours after birth

Which of the following diagnoses is most consistent with a bifid uvula in a newborn?

a. hypothyroidism
b. renal disease
c. cleft lip and palate
d. submucous cleft
d. submucous cleft

An enlarged tongue may indicate hypothyroidism in a newborn. Lip pits indicate cleft lip and palate. A two-vessel cord may indicate renal disease.

Urinary retention is a major cause of uterine atony which, in turn, permits which of the following?

a. excessive bleeding
b. constipation
c. digestive issues
d. hemorrhoids
a. excessive bleeding

A distended bladder lifts and displaces the uterus, making it difficult for it to remain contracted. Thus urinary retention is a major cause of uterine atony which, in turn, permits excessive bleeding. In addition, stasis of urine in the bladder predisposes the woman to urinary tract infection.

Serum Alpha-fetoprotein (AFP) is manufactured by the liver of the fetus and the mother. A high AFP might indicate which of the following?

a. Down syndrome
b. neural tube defect
c. fetal demise
d. maternal morbidity
b. neural tube defect

A high AFP might indicate neural tube defect. It can also result from multiple gestations. Down’s syndrome and fetal demise would be possibilities with a low AFP.

The cardiovascular system of the newborn begins to develop in the third week of gestation and is fully developed by the end of which week of gestation?

a. fifth week
b. sixth week
c. seventh week
d. eighth week
d. eighth week

The cardiovascular system is the first major organ system to develop in the embryo. Heart defects are among the most common birth defects and are the leading cause of birth defect related deaths.

All of the following are ways to keep a newborn from experiencing hypothermia EXCEPT

a. dry the baby thoroughly
b. do not bathe the baby for 48 hours
c. place a hat on the baby’s head
d. place skin to skin with mother
b. do not bathe the baby for 48 hours

Drying the baby thoroughly after birth, removing wet towels, placing a hat on the baby’s head, covering the scales with warm cloths and pre-warming clothes are the primary ways to fight heat loss. Not bathing the baby for 24 hours is optimal.

One of the most important causes of bleeding syndrome in an otherwise healthy newborn is hemorrhagic disease caused by deficiency of

a. vitamin A
b. vitamin B12
c. vitamin D
d. vitamin K
d. vitamin K

During the first week of life, newborns are at risk for bleeding disorders because of an immature liver that is unable to produce several coagulation factors and a sterile gastrointestinal tract that has not begun producing vitamin K.

A small lower jaw in a newborn may be an indication of which of the following pathologies?

a. facial nerve injury
b. Pierre Robin syndrome
c. teratogenic injury
d. esophageal atresia
b. Pierre Robin syndrome

A small lower jaw (micrognathia) can be indicative of Pierre Robin syndrome. It can also indicate Treacher Collins syndrome or De Lange syndrome.

Postpartum infection still accounts for significant rates of postpartum maternal morbidity and mortality. One type covers reproductive system infections. These are called

a. endometritis
b. mastitis
c. UTI
d. puerperal infections
d. puerperal infections

This is a bacterial infection that arises in the general tract after delivery. A woman is considered to have a puerperal infection if she has a fever of 38C (100.4F) or higher after the first 24 hours following delivery and the fever is maintained for at least 2 days within the first 10 days postpartum.

A newborn’s most important reflexes include all of the following EXCEPT

a. Moro
b. rooting
c. crying
d. sucking
c. crying

Practitioners look for unequal movement when assessing the reflexes of a newborn. Unequal movements could be a sign of an abnormality of the nerves. A newborn’s most important reflexes are the Moro, rooting, and sucking reflexes. Crying is not an important reflex in assessing a newborn.

Failure to diurese in the immediate postpartum period can lead to pulmonary edema and subsequent cardiac problems. This is seen more commonly in women with a history of

a. preeclampsia
b. diabetes
c. urinary tract infections
d. thyroid deficiency
a. preeclampsia

Diuresis in the first 2 to 5 days helps to decrease the extracellular fluid and results in a weight loos of 3 kg.

The fetal monitoring test which is a series of five assessments of fetal wellbeing is which of the following?

a. contraction stress test
b. Doppler ultrasound of umbilical artery
c. nonstress test
d. biophysical profile
d. biophysical profile

The biophysical profile (BPP) is a series of five assessments of fetal wellbeing, each of which is given a score of 0 (absent) or 2 (present). The parameters include a reactive nonstress test, the presence of fetal breathing movements, the presence of fetal movement of the body or limbs, the finding of fetal tone, and an adequate amount of amniotic fluid volume.

Nodules or plaques on the fetal surface of the placenta area associated with all of the following EXCEPT

a. infection
b. oligohydramnios
c. exposure to meconium
d. renal agenesis
c. exposure to meconium

Exposure to meconium may result in a green-stained placenta. All of the other conditions are associated with nodules or plaques on the fetal surface along with squamous metaplasia (benign).

Transient hypertension can occur during labor or immediately after delivery, but resolves within how many days?

a. 30 days
b. 21 days
c. 10 days
d. 5 days
c. 10 days

Newborns are most efficient in absorbing/digesting which of the following?

a. proteins
b. monosaccharides
c. carbohydrates
d. fats
b. monosaccharides

The newborn is particularly efficient in absorbing monosaccharides such as glucose, as long as the amount of glucose is not too large. Newborns are less able to digest proteins and fats than adults. Carbohydrate absorption is relatively efficient but still less than adult ability.

Often, a diagnosis for craniosynostosis can be done in which of the following ways?

a. measuring a baby’s skull
b. blood test
c. diagnostic imaging
d. genetic testing
a. measuring a baby’s skull

Further tests to confirm a diagnosis and check for related disorders include X-rays to check for fused (missing) sutures or ridges along sutures. Genetic tests to check for syndromes that cause craniosynostosis can be done. Diagnostic imaging, especially CT scans can be checked for fused sutures or ridges along sutures.

You are caring for a newborn infant boy who was preterm and has respiratory distress syndrome (RDS). In order to maximize that infant’s respiratory process the newborn should be placed in which position?

a. on his side in a flat position
b. in a supine position
c. in a prone position
d. on his side in a semi-Fowler position
c. in a prone position

Pacifier use for breastfed infants is recommended

a. once breastfeeding is well established by 3 to 4 weeks of age
b. immediately after birth
c. in the first 24 hours after birth
d. when mother’s nipples are sore
a. once breastfeeding is well established by 3 to 4 weeks of age

Early use of pacifiers may be associated with less successful breastfeeding.

After delivery, the mother’s weight decreases by approximately

a. 4 to 5 pounds
b. 10 to 12 pounds
c. 15 to 16 pounds
d. 20 pounds
b. 10 to 12 pounds

This weight loss is associated with the removal of the fetus, placenta, and amniotic fluid. An additional 5 pounds is lost during the early postpartum period as a result of diuresis and diaphoresis.

Newborns should regain their birth weight by 2 weeks of age and should continue to gain how much per week to be healthy?

a. 1 to 2 oz
b. 2 to 3 oz
c. 4 to 7 oz
d. 6 to 9 oz
c. 4 to 7 oz

Newborns should continue to gain 4 to 7 oz per week or at least 1 pound per month. An infant who has lost more than 7% of birth weight may have feeding problems that need to be addressed immediately.

In educating a new mother who is not able to breastfeed about bottle feeding, you would tell her all but which of the following?

a. the amount of formula that infant needs at 0 to 1 month is about 2-4 oz every 3 to 4 hours.
b. the amount of formula the infant needs at 2-4 months is about 5-7 oz every 4 to 5 hours.
c. suggest partial hydrolysate formula if the infant is at risk for cognitive deficiencies.
d. iron-fortified formula are best
c. suggest partial hydrolysate formula if the infant is at risk for cognitive deficiencies.

You would only suggest partial hydrolysate formula if the infant is at risk for atopic diseases, for instance a strong family history of atopic disease.

Diuresis in the postpartum woman begins within 12 hours after birth and continues for up to how long?

a. 72 hours
b. 5 days
c. 14 days
d. one month
b. 5 days

Diuresis occurs in response to the decrease in estrogen that stimulated fluid retention during pregnancy, the reduction of venous pressure in the lower half of the body, and the decrease in residual hypervolemia.

Severe iodine deficiency in the newborn is associated with which of the following?

a. Down syndrome
b. cretinism
c. Grave’s disease
d. cystic fibrosis
b. cretinism

Because normal development of the fetal neocortex is dependent on maternal T4, which is the primary source of cerebral T3, low levels of maternal T4 place the infant at risk for neurologic cretinism.

If you are testing a newborn’s hearing you would do which of the following tests?

a. auditory brainstem evoked response
b. Amsler test
c. Weber test
d. Rinne test
a. auditory brainstem evoked response

At 2 to 3 months, infants orient to voice, usually preferring the female voice.

Puerperal bradycardia commonly occurs during the first 6 to 10 days of the postpartum period. This may be related to all but which of the following?

a. decreased stroke volume
b. decreased cardiac strain
c. decreased blood volume
d. contraction of the uterus
a. decreased stroke volume

Increased stroke volume rather than decreased may be related to puerperal bradycardia. A pulse rate greater than 100 beats/min may be indicative of hypovolemia, infection, fear, or pain, and requires further assessment.

When testing a newborn’s oxygen levels with a pulse oximetry sensor, which of the following areas would be tested to detect congenital heart disease?

a. the newborn’s left hand and left foot
b. the newborn’s left hand and right foot
c. either hand and right foot
d. the right hand and either foot
d. the right hand and either foot

These structural heart defects are often associated with abnormally low levels of oxygen in the blood, known as hypoxemia, and can be a sign of a congenital heart disease.

The blue discoloration of newborn hands and feet is called

a. acrocyanosis
b. generalized cyanosis
c. central cyanosis
d. circumoral cyanosis
a. acrocyanosis

Generalized or central cyanosis may be seen initially at the time of birth and refers to the bluish color of the tongue, skin, lips, and nail beds in the newborn. Circumoral cyanosis is a bluish color seen around the newborn’s mouth.

Normal mucus production in the vagina usually returns when?

a. immediately after delivery
b. within 24 hours after delivery
c. with ovulation
d. with lactation
c. with ovulation

The vagina naturally changes after giving birth, and might feel wider, dry, or sore for some time.

The placenta requirements of the lungs are minimal in a full term fetus because its primary function is to support the

a. vascular pressures
b. blood flow
c. amniotic fluid
d. cardiac function
c. amniotic fluid

The placenta requires 40% of the fetal cardiac output per minute, with the lungs requiring 8-10%. The lung requirements are minimal because the primary function of the fetal lungs is to support the production of amniotic fluid, which continues until the time of birth.

Blood values should return to the pre-pregnant state at what point postpartum?

a. immediately after expulsion of the placenta
b. within 8 hours after birth
c. within 24 hours after birth
d. by the end of the postpartum period
d. by the end of the postpartum period

The postpartum period is commonly defined as the six weeks after childbirth.

In the fourth stage of labor, the uterus tone should be which of the following?

a. soft
b. firm
c. boggy
d. supple
b. firm

A soft, boggy uterus is not well contracted. Assess for postpartum hemorrhage and presence of blood or clots in the uterus if the uterus does not become firm following abdominal massage.

All of the cranial nerves, with the exception of the olfactory nerve, should be routinely assessed in the newborn. To assess the facial nerve (CN VII) which of the following would be assessed?

a. coordination of suck and swallow
b. head turns easily to either side
c. presence of gag reflex
d. symmetry of facial movement while crying or smiling
d. symmetry of facial movement while crying or smiling

Coordination of suck and swallow and presence of the gag reflex are assessments for the glossopharyngeal nerve (CN IX). Head turning easily to either side is an assessment for the accessory nerve (CN XI).

In the Apgar scoring system, if a newborn has blue, pale skin color, he would be given which of the following scores for appearance?

a. 0
b. 1
c. 2
d. 3
a. 0

Criteria for computing the Apgar score include appearance, pulse, grimace, activity, and respiratory effort. Each criterion is rated from 0 to 2.

The endometrium is regenerated by what point after giving birth?

a. within 72 hours
b. within 7 days
c. within 2-3 weeks
d. within 2-3 months
c. within 2-3 weeks

Immediately after delivery of the placenta, the placental site is approximately 8-10 cm, and by the end of the second week, it is about 3-4 cm.

In terms of pharmacology for the newborn, tachyphylaxis can be best described as which of the following?

a. a rapid decrease in drug response without a dosage change
b. an increase in serum concentrations higher than the recommended therapeutic range
c. the effect of pharmacokinetics on the newborn
d. an effect which takes place within the therapeutic range of the concentration of a drug
a. a rapid decrease in drug response without a dosage change

Puerperal bradycardia commonly occurs at which point in the postpartum period?

a. during the first 12 hours after birth
b. during the first 1 to 2 days postpartum
c. during the first 6 to 10 days postpartum
d. after breastfeeding has been initiated
c. during the first 6 to 10 days postpartum

It may be related to decreased cardiac strain, the decreased blood volume following placental separation, contraction of the uterus, and increased stroke volume.

A newborn who was born at 32 weeks gestation is in respiratory distress. Resuscitative efforts have been evaluated by the Silverman-Anderson Index of Respiratory Distress. The criteria graded include all but which of the following?

a. upper chest synchronization
b. lower chest retractions
c. inspiratory wheezing
d. Xiphoid retractions
c. inspiratory wheezing

The five criteria are: upper chest synchronization, lower chest retractions, Xiphoid retractions, nares dilation (flaring) and expiratory grunt. The lower the score on the Silverman-Anderson Index of Respiratory Distress, the better the respiratory status of the newborn.

Which of the following protects a new mother from excessive blood loss after the birth?

a. maternal platelets
b. lochia discharge
c. afterpains
d. maternal hypervolemia
d. maternal hypervolemia

Maternal hypervolemia typically occurs immediately following birth because the maternal circulation has an increase in blood volume that no longer travels through the placenta.

Recording the amount of lochia on the perineal pad by the hour is one method used for assessment of the postpartum woman. If the stain is approximately 6 inches in one hours, this typically indicates

a. scant
b. small
c. moderate
d. large
c. moderate

Approximately 6 inches of stain indicates a moderate amount of lochia; 2 inches indicates scant bleeding; 4 inches indicates small bleeding; over 6 inches indicates a large amount of bleeding. Institutions differ on the agreement of terms for measuring amounts of uterine discharge so review your agencies policy.

In terms of the sensory capabilities of a full term newborn which of the following is the least accurate statement?

a. newborns prefer black and white patterns
b. at term, the healthy newborn does not yet have an ability to fix on and track objects visually
c. the newborn has the ability to discriminate among distinctive odors
d. newborns have acute hearing and are able to localize sound in the environment
b. at term, the healthy newborn does not yet have an ability to fix on and track objects visually

Many studies have shown a strong newborn preference for patterns of stripes. During the first month of life, newborns become preferentially interested in patterns with contours that resemble the human face.

In a full term newborn, the heart rate may drop to a low of how many beats per minute during deep sleep?

a. 50
b. 50-60
c. 60-80
d. 80-100
d. 80-100

When a woman infected with Neisseria gonorrhea or Chlamydia trachomatis gives birth vaginally, which of the following can occur in the newborn?

a. ophthalmia neonatorum conjunctivitis
b. neonatal glaucoma
c. neonatal cataracts
d. retinopathy
a. ophthalmia neonatorum conjunctivitis

During the transitional period after birth, temperature, heart rate, rate and character of respirations, skin color, level of consciousness, muscle tone, and activity level are evaluated and documented at least once every 30 minutes until the newborn’s condition has remained stable for?

a. 1 hour
b. 2 hours
c. 4 hours
d. 8 hours
b. 2 hours

If the patient with amniotic fluid embolism is experiencing dyspnea and pulmonary edema, which of the following phases of AFE would it be considered?

a. Stage I
b. Stage III
c. Stage II
d. all of the above
a. Stage I

Dyspnea and pulmonary edema would be included in stage I of the phases of AFE. Other signs and symptoms which would be included in this phase would include: altered mentation, seizures, cardiac failure, left ventricular impairment, arrhythmias, acute renal failure, hypotension, shock, hypoxemia, respiratory acidemia, and cyanosis.

In documenting a patient’s medication, you inadvertently enter the wrong name for the medication. You immediately recognize the mistake. What is the most appropriate action to take?

a. draw a single line through the entry, initial it and enter the correct name for this medication
b. erase the error and enter the correct information immediately
c. use white-out to correct the error and enter the correct information immediately
d. report the mistake to the administration of the hospital
a. draw a single line through the entry, initial it and enter the correct name for this medication

Which of the following is NOT a goal of risk management?

a. decrease the number of claims
b. control costs related to claims
c. resolve patient problems through consultation
d. reduce the frequency of preventable injuries and accidents
c. resolve patient problems through consultation

The type of charting system that has a column-and-row format organized according to time and/or shift is which of the following?

a. narrative charting
b. flow sheet
c. problem-oriented charting
d. all of the above
b. flow sheet

Self determination is the basis for which of the following concepts in healthcare?

a. emancipation of minors
b. privacy
c. confidentiality
d. informed consent
d. informed consent

Self determination, also known as autonomy, is the philosophical basis for informed consent. Patients have the moral and legal right to determine what will be done to them.

Recommendations related to newborn identification and prevention of kidnapping have been developed by which of the following?

a. The Joint Commission
b. all state nursing boards
c. OSHA
d. the CDC
a. The Joint Commission

The minimum criteria by which proficiency is defined in the clinical area is which of the following?

a. ethical standards
b. standard of care
c. scope of practice
d. moral standards
b. standard of care

Which of the following questions would be the least important consideration in the observation of a newborn?

a. Are there any dysmorphic figures?
b. Are there any unusual rhythmical or jerking movements present?
c. Is the baby’s response to arousal normal for a neonate?
d. Is the baby breastfeeding?
d. Is the baby breastfeeding?

Eve is 2 days postpartum. During palpation, where would you normally expect to find her fundus?

a. at the level of the umbilicus
b. 2 fingers below umbilicus
c. 5 fingers below umbilicus
d. non-palpable at this time
b. 2 fingers below umbilicus

Keisha delivered her third child about 24 hours ago. She begins to c/o severe uterine cramping while breastfeeding. She is experiencing:

a. exfoliation of dead tissue
b. involution of uterus
c. shedding of the endometrium
d. after pains
d. after pains

Three days after delivering her child, Gretchen calls the 24 hour hotline. She is crying and states that her breasts suddenly became “bigger” and “really hurt.” She is experiencing:

a. colostrum production
b. a breast tumor
c. engorgement
d. overproduction of milk
c. engorgement

Tiffany delivered via cesarean 2 days ago. She is frustrated by constant perspiration and urination. This is likely the result of:

a. changes in urine pH and a possible UTI
b. drinking too much fluid in the early postpartum period
c. the body’s removal of fluids accumulated during pregnancy
d. receiving excessive amounts of IV fluids during labor
c. the body’s removal of fluids accumulated during pregnancy

While assessing Sierra, a 32yo mother of 3, you notice uterine bleeding and find that her uterus is displayed upward and to the right. This is probably cause by:

a. retained placental fragments
b. rupture of the uterus
c. distention of the bladder
d. cervical bruising
c. distention of the bladder

Olga, a 38yo patient whom you administered the rubella vaccine, states that at her age she plans to get pregnant right away. You tell her she should:

a. avoid pregnancy for at least 8 weeks after receiving the MMR vaccine
b. get pregnant ASAP because of her age
c. avoid getting pregnant for at least a year after receiving the MMR vaccine
d. abstain from sex for 2 weeks after delivery
a. avoid pregnancy for at least 8 weeks after receiving the MMR vaccine

Joan is a 17yo and just delivered a healthy baby boy 6 hours ago. She talks on her cell phone whenever you are in the room and refuses to touch or hold her baby, even when he cries. You overhear Joan tell someone that the baby is “ugly like his father.” Joan is most likely experiencing:

a. exhaustion
b. mal-attachment
c. normal attachment
d. postpartum depression
b. mal-attachment

Recommended advice for breastfeeding mothers include:

a. avoid the use of a sports bra
b. wash the nipples with soap and water after each feeding
c. apply a warm compress to ease nipple pain
d. “conditioning” the nipples by gently rubbing them with moist tea bags
c. apply a warm compress to ease nipple pain

Samantha read in the discharge booklet that postpartum patients are at risk for thrombophlebitis, and she asks you how she can prevent this from happening. You instruct her to:

a. remain on strict bedrest
b. ambulate frequently
c. cross her legs while sitting
d. avoid elevating her legs
b. ambulate frequently

Suzanne delivered vaginally 8 hours ago. She has uterine cramping and rates her pain a “9” but refuses pain medication. What is your best response regarding her pain?

a. tell her she should take the ibuprofen as prescribed by her MD
b. encourage her to fight the pain because it will make her stronger
c. ask her to use the call button if she changes her mind about taking pain medication
d. offer her a non-drug option such as a hot pack or massage
d. offer her a non-drug option such as a hot pack or massage

Emily is a 28yo mother of 3 who delivered vaginally 4 hours ago. She reports she is “bleeding a lot” and her perineal pad, changed 45 minutes ago, is fully saturated. Palpation reveals a boggy uterus. Your first intervention is:

a. start IV fluids
b. place a catheter
c. massage her uterus until firm
d. call the MD
c. massage her uterus until firm

Mei experienced premature rupture of membranes during her pregnancy and her internal fetal monitors placed during labor. She later delivered by cesarean. 12 hours after delivery, her HR is 104 and her lochia smells foul. You suspect:

a. endometriosis
b. mastitis
c. uterine cancer
d. endometritis
d. endometritis

You are preparing to discharge Maria after delivering a healthy boy. You believe Maria understands the discharge teaching instructions when she states:

a. it is acceptable to have bleeding nipples after breastfeeding for the first few days
b. I should not worry if I have a large mass under my breasts
c. I will call my nurse-midwife if I saturate a pad in one hour
d. suicidal thoughts are okay because the baby blues are normal
c. I will call my nurse-midwife if I saturate a pad in one hour

An infant born to a mother who abused drugs during pregnancy exhibited withdrawal signs in the nursery that included fever and poor state control. After stabilization and discharge, the caretaker called to report a return of withdrawal symptoms. This pattern is consistent with maternal abuse of:

a. alcohol
b. cocaine
c. methadone
b. cocaine

Leukocytosis that occurs in normal postpartum adaptation can be differentiated from an infectious process by the presence of:

a. a 30% increase in WBC count in a 6 hour period
b. a concominant increase in the sedimentation rate
c. any elevation in temperature
b. a concominant increase in the sedimentation rate

Tachypnea, tachycardia, and poor feeding in a 12 hour old neonate, born to a mother with diabetes mellitus, would be more consistent with the onset of:

a. congestive heart failure
b. hypoglycemia
c. respiratory distress syndrome
b. hypoglycemia

A postpartum woman has a BP of 150/92 at 6 hours. Later, the BP is 148/90. This finding most likely:

a. diminished venous return
b. reflects postpartum peak in cardiac output
c. requires evaluation for postpartum preeclampsia
c. requires evaluation for postpartum preeclampsia

The let down reflex in the breastfeeding woman is responsible for

a. movement of the milk from ducts to the nipple
b. regulation of milk release to ensure milk is available at any given time
c. release of the milk to the alveoli
a. movement of the milk from ducts to the nipple

A woman plans not to breastfeed and asks about how to suppress lactation. She should be advised that:

a. mechanical suppression occurs with the use of a supportive bra
b. medications can be prescribes to stop lactation
c. physiologically, lactation will not proceed without breastfeeding and no intervention is needed
a. mechanical suppression occurs with the use of a supportive bra

Spinal blockade occurring with epidural anesthesia results in

a. hypotension
b. tachycardia
c. vertigo
a. hypotension

Synthesis of vitamin K in the neonate is dependent upon the presence of

a. amino acids derived from human or formula proteins
b. appropriate bacteria in the GI tract
c. passively acquired maternal antibodies
b. appropriate bacteria in the GI tract

A woman with a history of breast augmentation asks if she will be able to breastfeed her baby. The factor that will most influence her ability to breastfeed is

a. time interval since the surgery
b. type of implant
c. type of incision
c. type of incision

Assessment for the presence of congenital viral infection would be indicated in which of the following:

a. 38 weeks, LGA infant with RDS
b. late preterm with elevated indirect bilirubin
c. term, SGA with widespread petechiae
c. term, SGA with widespread petechiae

A woman has been treated with fractionated heparin during pregnancy for thromboembolism. She wants to breastfeed. In the postpartum period, it would be expected that she would

a. be changed to a low molecular weight heparin formulation
b. be switched to warfarin
c. continue with heparin therapy
c. continue with heparin therapy

An anatomic breast variation that has been linked with insufficient lactation relates to

a. inadequate glandular tissue
b. lack of breast tissue elasticity
c. shape and size of nipple
a. inadequate glandular tissue

A woman is admitted to the postpartum unit following delivery of her infant on the route to the hospital. The RN noted bruises on the woman’s back and arms. To assess possible intimate partner violence, the RN should

a. directly ask her if anyone has threatened her or physically harmed her
b. question her on how she obtained the bruises
c. review the patients chart for any indication of domestic violence
b. question her on how she obtained the bruises

The use of phototherapy reduces the risk of

a. bronze baby syndrome
b. exchange transfusion
c. skin moles
b. exchange transfusion

Uterine prolapse associated with childbirth is due to

a. atrophy of the uterine tissue
b. bulging of uterine tissue through the cervix
c. weakening of the pelvic tissue
c. weakening of the pelvic tissue

A new mother verbalizes her concern about the presence of stork bites nevus on her infant’s eyelid. The RN’s response will be based on the knowledge that

a. further growth and facial involvement is likely overtime
b. the location of the birth mark carries a risk of eye involvement
c. this type of vascular lesions tend to disappear by the age of one
c. this type of vascular lesions tend to disappear by the age of one

The recommended antibiotic treatment for postpartum endometritis is

a. a broad spectrum antibiotic
b. based on endometrial culture
c. selected based on provider’s preference
a. a broad spectrum antibiotic

It is difficult to determine the extent of blood loss with a vaginal hematoma because

a. blood is retained in the tissue
b. blood loss is mistaken as part of the normal postpartum course
c. symptoms don’t correlate to the amount of blood loss
a. blood is retained in the tissue

A newborn is noted at birth to be hypotonic with characteristics that include small ears, flat appearance to the face, slanted palpebral tissues, and simian creases on both palms. Based on the findings, the RN can expect a work up that includes

a. blood work sent for TORCH titers
b. genetic consult from chromosome studies
c. review of the prenatal history of maternal alcohol use
b. genetic consult from chromosome studies

Milk transfer, a key physiologic principle in lactation, is most efficient with

a. adequate prolactin levels
b. good lactation
c. strong negative pressure action by the baby
b. good lactation

More than 2/3 of sentinel events reported to the Joint Commission were caused by

a. communication failure among healthcare team members
b. fatigue and shift work related problems
c. inadequate staffing
a. communication failure among healthcare team members

During the recovery period following a rapid delivery, a woman had an elevated BP, epigastric pain, and severe headache. IV mag sulfate was started at 2 gm/hr. 2 hours later, the woman is lethargic with worsening visual disturbances and has now lapsed into a comatose state. She should be evaluated for

a. cerebral hemorrhage
b. HELLP syndrome
c. mag toxicity
c. mag toxicity

A central venous hematocrit would be chosen over a heel stick sample to confirm diagnosis of

a. hyperbilirubenemia
b. polycythemia
c. vitamin K deficiency
b. polycythemia

Polycythemia occurs more frequently in the IUGR neonate due to

a. decreased erythropoietin activity
b. immaturity of the hematologic system
c. physiologic response to hypoxia
c. physiologic response to hypoxia

Which of the following would be suggestive of a hydrocele?

a. a bluish-red colored swelling of the scrotum
b. irreducible swelling of the scrotum that transilluminates
c. presence of scrotal mass which does not transilluminate
b. irreducible swelling of the scrotum that transilluminates

A breastfeeding woman developed endomyometriosis 3 days after her cesarean. She was treated with IV antibiotics and discharged home on oral antibiotics. On her 6th postpartum day, she returns to the hospital with fever, shaking chills, tachycardia, and flank pain. She has urinary frequency and urgency. The patient should be further assessed for

a. cystitis
b. pyelonephritis
c. wound infection
b. pyelonephritis

Assessment of which of the following neonatal reflexes would be most important in evaluation for brachial plexus nerve damage?

a. moro
b. rooting
c. stepping
a. moro

A woman with preeclampsia is at greater risk for an eclamptic seizure if she has

a. developing epigastric abdominal pain
b. elevated BP despite being treated with mag sulfate
c. poor urinary output
c. poor urinary output

The greatest morbidity from placental abruption occurs in women who

a. develop malnutrition
b. develop preeclampsia with a preterm gestation
c. have a history of preeclampsia with previous pregnancy
b. develop preeclampsia with a preterm gestation

The breastfeeding woman with a yeast infection on the breast, typically presents with

a. breast pain while breastfeeding
b. development of generalized rash of the breast
c. low grade fever that doesn’t respond to antipyretics
a. breast pain while breastfeeding

Release of hepatic glucose is a response that occur in term neonates

a. at times of decreased glucagon secretion
b. following initial attempt of breastfeeding
c. with declining glucose levels following birth
c. with declining glucose levels following birth

The newborn experiencing a delay in clearing fetal lung fluid after birth is at increased risk for

a. aspiration pneumonitis
b. RDS
c. transient tachypnea of the newborn
c. transient tachypnea of the newborn

A method to differentiate between jitteriness and potential seizure activity is to

a. passively flex the extremities to determine if tremors cease
b. make a loud noise to see if tremors are elicited
c. note the length of time that tremor activity continues
a. passively flex the extremities to determine if tremors cease

A 4 hour old newborn that has been swaddled and sleeping soundly for about 2 hours is noted to have a HR of 170/min. An appropriate initial intervention would be to

a. contact the PCP immediately
b. obtain an axillary temperature
c. prepare for a sepsis workup
b. obtain an axillary temperature

A mother asks how her newborn’s vision can be evaluated in the nursery. The RN responds by

a. demonstrating that the baby can fix on and follow a bright object
b. explaining that the vision cannot be assessed because of eye prophylaxis
c. holding the baby upright to show that the baby’s eyes open
a. demonstrating that the baby can fix on and follow a bright object

Hypercoagulability in pregnancy places the postpartum woman at an increased risk for

a. DVT
b. greater blood loss at delivery
c. polycythemia
a. DVT

Maternal mag sulfate administration during labor places the neonate at increased risk for

a. cardiac dysrhythmias
b. decreased suck reflex
c. GI reflux
b. decreased suck reflex

A normal finding on inspection of the perineum following suturing of a laceration 24 hours postpartum is

a. presence of ecchymoses
b. skin edges that are not fully approximated
c. some edema or tenderness
c. some edema or tenderness

Postpartum psychosis is a variant of

a. bipolar disorder
b. an anxiety disorder
c. PTSD
a. bipolar disorder

A priority of care to administering methergine postpartum is to check

a. blood pressure
b. history of asthma
c. urinary output
a. blood pressure

Vaginal examination is initiated for a woman who reports increasing episiotomy pain to detect

a. hematoma
b. retained placental fragments
c. undetected laceration
a. hematoma

An otherwise stable 12 hour old term neonate is noted to have multiple white topped papules on reddened macular bases on the face and trunk. Based on these findings, the RN would recognize the need to

a. document as a benign finding
b. institute airborne isolation ASAP
c. obtain a surface culture
a. document as a benign finding

A parameter used in Ballard score for determination of gestational age is

a. babinski reflexes
b. hair texture
c. heel to ear maneuver
c. heel to ear maneuver

The primary function of the ductus arteriosis in the normal fetus is to

a. ensure circulation to the liver
b. provide a route for blood to bypass the lungs
c. shunt blood to the right side of the heart
b. provide a route for blood to bypass the lungs

Insulin requirements decrease in the immediate postpartum period, in a diabetic woman, because

a. anti-insulin placental hormones decrease
b. exogenous insulin production increases
c. moderate ketonuria develops
a. anti-insulin placental hormones decrease

The major challenge of the adolescent mother transitioning to the role of motherhood is

a. egocentricity
b. lack of life experience
c. physiologic immaturity
a. egocentricity

Increased stroke volume during the postpartum period is responsible for what normal physiologic postpartum change?

a. increased cardiac output
b. increased heart rate
c. reduced vascular resistance
a. increased cardiac output

Attention to the neonate’s age in hours is essential when

a. determining the timing of metabolic screening
b. interpreting total serum bilirubin levels
c. scheduling a circumcision
b. interpreting total serum bilirubin levels

Three hours following a prolonged labor that resulted in a cesarean, a postpartum woman shows little interest in her infant and is reluctant to breastfeed. The reason for this behavior is most likely related to

a. possible attachment disorder
b. extreme fatigue
c. postpartum depression
b. extreme fatigue

Once breastfeeding is well established, milk production is sustained by

a. adequate maternal calorie and fluid intake
b. consistent milk removal by breastfeeding or pumping
c. increasing prolactin production
b. consistent milk removal by breastfeeding or pumping

At a postpartum home visit, a woman who had a cesarean 2 weeks ago reports tenderness, edema, and skin discoloration. She is afebrile and now has drainage. It is important to recognize that

a. fever must be present for a diagnosis of infection
b. these are early signs of infection and should be evaluated
c. the presents the normal healing process
b. these are early signs of infection and should be evaluated

The mother of a 2 day old infant, recently discharged, calls the nursery in a panic because the diaper has a streak of rust colored staining. The RN’s response to this mother will be based on the knowledge that this finding is

a. a normal reaction between urine and urates
b. an early sign of renal failure
c. possible hematuria and suggests a bleeding disorder
a. a normal reaction between urine and urates

The fetus of a diabetic mother has a lecithin/sphingomyelin (L/S) ratio of 2:1. It should be understood that

a. the fetus will need surfactant
b. there is a high rate of false positive findings in this population
c. this is a definitive finding of lung maturity in any fetus
b. there is a high rate of false positive findings in this population

For the late preterm neonate being bottle fed, a supportive feeding intervention is to

a. enlarge the nipple hole for better milk flow and less fatigue
b. insert the nipple and use a push-pull motion to stimulate suck
c. use a low flow nipple and finish the feeding by gavage if needed
b. insert the nipple and use a push-pull motion to stimulate suck

A neonate has experienced multiple heel sticks to follow bilirubin levels. The RN notes during morning lab stick that the infant has a fixed gaze and no response to the heel stick. This observation is indicative of

a. a mature neonatal behavior state
b. decreased responsiveness to a repeated stimulus
c. the onset of acute bilirubin encephalopathy
b. decreased responsiveness to a repeated stimulus

A rational for measuring head circumference at birth is that it serves as a reflection of

a. birth trauma experienced
b. brain growth during gestation
c. intrauterine positioning effects
b. brain growth during gestation

The newborn’s ability to concentrate urine is reflected in results of a

a. 24 hour urine output total
b. serum potassium level
c. urine specific gravity reading
c. urine specific gravity reading

The most common complication of mastitis is

a. cessation of breastfeeding
b. development of breast abscess
c. systemic infection
b. development of breast abscess

A postpartum effect of oxytocin administration during labor is

a. hypertension
b. hypocalcemia
c. increased lochia flow
a. hypertension

The RN would suspect the presence of a patent urachus in the newborn who has

a. large edematous umbilical cord that remains moist
b. staining of the cord with meconium flecks around the base
c. visible loops of bowel within the Wharton’s jelly
a. large edematous umbilical cord that remains moist

Leakage of spinal fluid at the puncture site during administration of spinal anesthesia can lead to

a. headache
b. meningeal inflammation
c. transient motor dysfunction
a. headache

A term neonate is born with gasping respirations and HR of 50/min following an uncomplicated birth. By 30 seconds of age, initial steps of resuscitation are ongoing use bag/mask ventilation. However, the neonate remains apneic with a HR of 65/min. The most appropriate intervention at this point is

a. administer epinephrine
b. continue positive pressure ventilation
c. initiate cardiac compression
c. initiate cardiac compression

A redistribution of cardiac output to the heart and brain occurs during

a. periods of aerobic metabolism
b. prolonged metabolic alkalosis
c. the onset of asphyxiation
c. the onset of asphyxiation

A neonate is born at 36 weeks following a placental abruption. At 2 hours of age, the infant has a BP of 88/57. These findings are most consistent with

a. alcohol
b. cocaine
c. methadone
b. cocaine

Butarphanol (Stadol) should not be given to a drug dependent woman because of the risk of

a. profound respiratory depression
b. seizures
c. withdrawal symptoms
c. withdrawal symptoms

Which of the following descriptions best characterizes a term neonate who experienced IUGR?

a. birth weight of 2489g with the facial appearance of an old man
b. head circumference of 32.5cm, with reduced lanugo over the body
c. length of 49cm with vernix present only on major body creases
a. birth weight of 2489g with the facial appearance of an old man

As the transition of colostrum to mature milk occurs, there is a decrease in

a. fat content and calories per mL of milk
b. immunoglobulin concentration
c. water content
b. immunoglobulin concentration

At birth, a physiologic change that occurs in the neonate once respirations are initiated is

a. decreasing pulmonary vascular resistance
b. falling alveolar PO2 levels
c. increasing alveolar surface tension
a. decreasing pulmonary vascular resistance

A primary reason for using erythromycin or a tetracycline eye ointment instead of silver nitrate for newborns is

a. silver nitrate is not effective against chlamydia
b. there is a shortage of the silver nitrate
c. use of silver nitrate masks the signs of conjunctivitis
a. silver nitrate is not effective against chlamydia

A soft swelling confined to the parietal area of the skill is observed in an 18 hour old neonate born vaginally following prolonged labor. This presentation is consistent with the presence of

a. caput succedaneum
b. cephalahematoma
c. subgaleal hemorrhage
a. caput succedaneum

When caring for parents who have suffered a perinatal loss, it is important to remember that the grief response

a. is abnormal if emotions are not immediately expressed
b. is individual to each parent
c. follows the order of the 5 stages of grief
b. is individual to each parent

A breastfeeding woman reports pain during breastfeeding because the baby is chewing on her nipple. This represents a problem related to

a. breast anatomy
b. insufficient milk supply
c. latch
c. latch

Craniotabes can be best described as

a. an elongated head with flattened temporoparietal regions
b. premature fusions of one or more sutures
c. softened skull bones that indent and pop back into place
c. softened skull bones that indent and pop back into place

An RN realizes that during the previous day’s shift, a call to the attending physician to report a critical lab value was not charted. Since hospital policy was followed in making the call and appropriate follow up was done, at this point the RN should

a. avoid any additional documentation
b. complete an occurrence report for submission to risk management
c. enter the info as late entry with current date and time indicated
c. enter the info as late entry with current date and time indicated

The most common cause of fever during the early postpartum period is

a. breast engorgement
b. endometriosis
c. UTI
b. endometriosis

A mother contacts the nursery because her 1 week old term infant “has started to turn yellow.” The RN should recognize that the onset of jaundice in this infant is most likely a result of

a. antibody-induced hemolysis
b. maternal hormones in the breast milk
c. physiologic hepatic function
b. maternal hormones in the breast milk

Low levels of estrogen in the postpartum period have what effect?

a. decrease in ovarian function
b. diaphoresis
c. transient urinary incontinence
a. decrease in ovarian function

Based on patterns of fetal and neonatal glucose metabolism, the nurse would monitor the glucose status of an infant born to a mother with DM because of the anticipated impact of

a. decreased hepatic glycogen stores
b. increased insulin responses
c. sudden onset of glycogenesis
a. decreased hepatic glycogen stores

A test performed to detect classic developmental dislocation of the hip is

a. barlow
b. ortolani
c. pavlik
b. ortolani

A predisposing factor of retained placental fragments is

a. manual removal of the placenta
b. multiple gestation
c. placenta accreta
c. placenta accreta

Placement of an orogastric tube during initial stabilization of a neonate with suspected duodenal atresia is done primarily to

a. decompress the stomach
b. remove excess fluid
c. prevent GI reflux
a. decompress the stomach

A breastfeeding woman at 2 weeks postpartum is pumping her breasts to increase her milk supply. She should be advised that the factor that will most stimulate milk production when pumping is

a. duration of each pumping session
b. frequency of pumping sessions
c. use of an electric pump
b. frequency of pumping sessions

A postpartum woman with preeclampsia remains on magnesium after delivery. It is important for the RN to monitor urine output because magnesium can cause

a. decreased creatinine clearance
b. oliguria
c. osmotic diuresis
c. osmotic diuresis

What is means to be a father for most men is

a. affected by how he was fathered
b. determined by financial readiness to assume the role
c. determined by how well he prepared for the role during pregnancy
c. determined by how well he prepared for the role during pregnancy

In the absence of vascular disease, the pathological manifestation of DM in pregnancy is usually the result of

a. hyperglycemia
b. hyperinsulinemia
c. insulin resistance
c. insulin resistance

Oligohydramnios associated with PROM in preterm pregnancies at less than 24 weeks may cause

a. bronchopulmonary dysplasia
b. pulmonary hypoplasia
c. surfactant inactivation
b. pulmonary hypoplasia

A woman with type II diabetes is breastfeeding. She is on insulin therapy. To minimize hypoglycemic episodes, she should be advised to

a. discuss using oral agents while breastfeeding with her MD
b. make sure she has a snack with each breastfeeding session
c. use a split dose insulin regimen
b. make sure she has a snack with each breastfeeding session

A woman is recovering from the birth of her child which was complicated by placental abruption and postpartum hemorrhage. She is pale, clammy, hypotensive, and tachycardic. These findings are consistent with

a. DIC
b. hereditary thrombophilia
c. Sheehan’s syndrome
a. DIC

The hypoestrogenic state in the postpartum period is responsible for

a. a delay in the perineal healing
b. development of cervical edema
c. dyspareunia
c. dyspareunia

A finding noted during an initial newborn physical exam that warrants further follow up would be the presence of

a. small area of midline swelling in the back
b. diastasis rectus of the abdominal musculature
c. small white inclusion cysts along the gum line
a. small area of midline swelling in the back

The RN would anticipate that a term newborn with a CV condition marked by right to left shunting through the foramen ovale would appear

a. cyanotic
b. pink
c. very pale
a. cyanotic

The dilation of the ureters and renal pelvis that occurs during pregnancy reverts at what time?

a. between 2-8 weeks postpartum
b. does not occur until after uterine involution
c. never completely reverses but is nearly normal by 6 months postpartum
c. never completely reverses but is nearly normal by 6 months postpartum

Vitamin K preparation for neonatal prophylaxis should be

a. given with 4 hours
b. diluted prior to subQ administration
c. stored away from light
c. stored away from light

A newborn is ready for discharge following a course of phototherapy for ABO incompatibility. Based on the history, the RN will convey the important of keeping the infant’s scheduled follow up appointment because of the increased risk of

a. dehydration
b. late onset hearing loss
c. rebound bilirubin levels
c. rebound bilirubin levels

A woman with unilateral mastitis can be advised that she can continue to breastfeed

a. if there is no abscess present
b. with the use of nipple shields on the affected breast
c. without restrictions
a. if there is no abscess present

A 2 hour old newborn appears generally pale with a dusky appearance to the nail beds and a grayish appearance to the mucous membranes inside the mouth. The finding most indicative of oxygen desaturation is the presence of

a. dusky nail beds
b. grayish mucous membranes
c. overall pale
b. grayish mucous membranes

The uterus of a postpartum woman is boggy and midline. Initial management should include

a. fundal massage
b. having her void
c. vaginal exam
a. fundal massage

On uterine assessment, the RN notices that the uterus is boggy and difficult to differentiate. While she massages it with one hand, she places one hand just above the symphysis pubis in order to

a. assess the bladder distention
b. minimize lochia flow
c. provide stability to the uterus
c. provide stability to the uterus

A 2 day old neonate born at 36 weeks develops jaundice and is breastfeeding poorly. In addition to a moderate evaluation in the total serum bilirubin, lab results indicate O Rh positive blood type with a maternal blood type A Rh positive. The RN would recognize that a major contributor to the rising bilirubin level is most likely

a. increased enterohepatic shunting
b. ongoing hemolysis
c. presence of maternal hormones in the breast milk
b. ongoing hemolysis

An RN would provide pre-discharge education regarding the need to assess a neonate for jaundice if the mom received drug therapy within a few weeks of birth using medications classified as

a. anticoagulation
b. antihistamines
c. sulfonamides
c. sulfonamides

A predisposing factor to amniotic fluid embolism is

a. adolescent pregnancy
b. distocia
c. induction of labor
b. distocia

The newborn with documented polycythemia should be further assessed for jaundice based on

a. a genetic enzyme deficiency
b. an increased production of bilirubin
c. the presence of a systemic infection
b. an increased production of bilirubin

The increase in hematocrit that occurs in the 1st postpartum week is due to

a. decreased plasma volume
b. erythrocytosis
c. increased platelet aggregation
a. decreased plasma volume

Assessment of upper and lower extremity BP would be an appropriate initial response to a finding of

a. bounding brachial pulses
b. capillary refill time of 6 seconds
c. diminished or absent femoral pulses
a. bounding brachial pulses

Reversal of warfarin is best achieved by

a. prostigmin
b. protamine sulfate
c. vitamin K
c. vitamin K

Two hours after a cesarean, a woman is hypotensive and has difficulty breathing with cyanosis present consistent with

a. adult RDS
b. amniotic fluid embolism
c. pulmonary embolism
c. pulmonary embolism

A trigger of the breast milk ejection reflex can be induced by the woman when she

a. experiences extreme temperature fluctuations
b. initiate a change in diet or drinking habits
c. thinks of the baby or hears the baby cry
c. thinks of the baby or hears the baby cry

A woman at 3 days postpartum has developed very engorged breasts. This is most likely due to

a. excessive milk production
b. inadequate milk removal
c. poor vascularity of breast tissue
b. inadequate milk removal

The most sensitive sign of pulmonary embolism is

a. chest pain
b. unexplained tachycardia
c. unremitting cough
a. chest pain

Use of the ventral suspension maneuver is useful in assessing muscle tone of the

a. extremities
b. neck
c. trunk
b. neck
c. trunk

A blood patch is planned as treatment for a post epidural headache. The RN accompanies the MD for the explanation to the patient as to how the procedure will be done, how the process works in relieving the pain, and how fast pain relief can be expected. In terms of the consent process, the

a. informed consent process has been legally completed
b. RN can now answer any further questions that might arise about the procedure
c. required discussion of potential risks and alternative therapies was omitted
c. required discussion of potential risks and alternative therapies was omitted

At birth, a neonate has no signs of respiratory distress when crying, but develops cyanosis and dyspnea when quiet. An intervention indicated for this infant is

a. an emergent surgical procedure
b. intubation in the delivery room
c. placement of an oral airway
c. placement of an oral airway

In the term neonate under stress, thermogenesis occurs primarily from

a. calories produced by random muscle activity
b. heat output attained from shivering
c. metabolism of brown adipose tissue
c. metabolism of brown adipose tissue

A woman is now 2 hours postpartum following birth by vacuum extraction of a 3200g baby. Vital signs are BP 89/79, HR 120, and temp 98.6. The uterus is firm, but there is a steady stream of bleeding without clots. The most likely cause of these finding is

a. cervical laceration
b. retained placenta
c. uterine inversion
a. cervical laceration

A woman with a history of a positive culture for strep B was admitted in active labor with intact membranes at 36 weeks gestation. She received IV ampicillan 2 hour prior to a vaginal birth. The newborn has Apgar scores of 9/9 and a normal physical exam. The most appropriate management approach is to

a. follow well baby nursery routines
b. initiate therapy with IV antibiotics
c. obtain a complete blood count and blood cultures
c. obtain a complete blood count and blood cultures

At birth, a SGA baby is noted to have microcephaly, a recessed chin, upturned nose and sunken nasal bridge. These findings are characteristics of

a. congenital CMV infection
b. fetal alcohol syndrome
c. 1st trimester rubella infection
b. fetal alcohol syndrome

An antibody screen for an Rh negative postpartum woman is negative and so is the Coombs test on the neonate. This requires that Rh immunoglobulin

a. be administered at her first postpartum visit
b. be given at discharge from the hospital
c. should be withheld
b. be given at discharge from the hospital

A woman 4 hours postpartum talks frequently and in detail about what happened during labor and the birth of the baby. The RN should recognize this is a sign of

a. anxiety from transition to the maternal role
b. normal postpartum adjustment
c. stress from experience of labor and delivery
b. normal postpartum adjustment

A breastfeeding woman has a plugged duct. To address this problem, the RN should recommend

a. cold compresses to the area
b. heat to the area
c. increasing fluid consumption
b. heat to the area

A new mother asks about letting her baby’s umbilical cord dry naturally instead of using alcohol applications. The RN should advise that natural drying

a. is safe and acceptable
b. lengthens the time for cord separation
c. predisposes for infection
a. is safe and acceptable

When finding of clinical studies are not likely to have occurred by chance alone, the results are said to be

a. appropriate for publication as guidelines
b. ready for use in general clinical areas
c. statistically significant in research terminology
c. statistically significant in research terminology

Following an uncomplicated pregnancy, a woman delivers a stillborn infant at term. In caring for the family, the RN can anticipate the most likely reaction to the unanticipated loss is

a. disbelief or protest
b. feeling of guilt
c. withdrawal behavior
b. feeling of guilt

A neonate is diagnosed with trisomy 21. The baby is prone to have

a. bowel obstruction
b. congenital heart defect
c. open neural tube
b. congenital heart defect

The presence of peripheral cyanosis in the neonate reflects

a. increased tissue oxygen extraction
b. low circulating blood volume
c. vascular compromise
c. vascular compromise

A woman receives an epidural during labor and delivery. In the postpartum period, she is at increased risk for

a. hypotension
b. loss of motor control
c. urinary retention
c. urinary retention

In the life threatening event of amniotic fluid embolism, standard management includes immediate

a. attention to maternal positioning
b. bilateral uterine artery embolization
c. surgical removal of uterus
a. attention to maternal positioning

When a conflict arises between an RN and PCP regarding non-emergent management of a patient, the first step in conflict resolution is

a. document the particulars in the patient’s chart
b. have a conversation with the PCP about the issue
c. notify the RN manager of the situation
c. notify the RN manager of the situation

A well term baby has a glucose reading of 44 at 72 hours of age. This is considered

a. low and needs intervention
b. physiologic hypoglycemia
c. WNL
c. WNL

Use of the pull-to-sit maneuver is useful in

a. assessing postural tones
b. eliciting the tonic neck reflux
c. testing for truncal incurvation
a. assessing postural tones

A distinguishing finding of perineal laceration is

a. firm fundus with bright red bleeding
b. persistent lochia rubra
c. vaginal pain with little bleeding
a. firm fundus with bright red bleeding

How a fetus is positioned in utero can affect

a. brain growth
b. head symmetry
c. lung volume
b. head symmetry

Maternal mag sulfate administration during labor places the neonate at increased risk for

a. CNS depression
b. hyperbilirubinemia
c. poor feeding
a. CNS depression

Post-spinal anesthesia headache is a result of

a. effects of the blockage of sympathetic nerve fibers
b. leakage of the cerebrospinal fluid
c. meningeal irritation
b. leakage of the cerebrospinal fluid

Phototherapy yields the best results in decreasing bilirubin levels when

a. blue light spectrum lights are used
b. the infant is frequently repositioned during therapy
c. there is maximum skin exposure
c. there is maximum skin exposure

The newborn’s limitation of tubular reabsorption predisposes it to an

a. excessive insensible water loss
b. high specific gravity
c. loss of bicarbonate
a. excessive insensible water loss

A 4 hour old baby has been swaddled and sleeping soundly for about 2 hours and is now noted to have a sustained HR of 170/min. VS WNL. The next step in evaluation is

a. arterial blood gas
b. echocardiogram
c. electrocardiogram
c. electrocardiogram

A woman who is HIV positive wants to breastfeed. She should be advised that

a. breastfeeding is contraindicated
b. the decision to breastfeed will be based on her viral load count
c. the type of antiviral therapy she is taking will determine if breastfeeding can be initiated
a. breastfeeding is contraindicated

What sleep state is characterized by variable activity level with smooth movements, occasional startles, and eyes that open and close but appear dull and heavy?

a. drowsiness
b. light sleep
c. quiet alert
a. drowsiness

A term neonate develops bronze baby syndrome while under phototherapy. This results from

a. concurrent presence of cholestatic jaundice
b. irradiance exposure
c. melanin breakdown in the skin
a. concurrent presence of cholestatic jaundice

The underlying cause of GDM is

a. CHO intolerance
b. diminished glucose transport
c. insulin deficiency
a. CHO intolerance

Compared to vaginal deliveries, the woman who delivers by cesarean will have what different lochia characteristics?

a. a lesser amount
b. heavier lochia on standing
c. longer time to achieve lochia alba stage
a. a lesser amount

For optimal evaluation response to visual stimuli, the neonate should be

a. assessed in the quiet alert state
b. placed in a sitting position
c. tested for close, mid-range, and far away visual distances
a. assessed in the quiet alert state

The constellation of symptoms of facial abnormalities, growth restriction, and brain damage that leads to intellectual difficulties is consistent with

a. fetal alcohol syndrome
b. trisomy 13
c. varicella infection
a. fetal alcohol syndrome

A neonate has experienced multiple heel sticks. The RN notes that infant no longer cries or has any response. This change in response is most likely related to

a. behavioral stress response to repeated stimuli
b. overstimulation of pain receptors
c. possible nerve damage in the foot
a. behavioral stress response to repeated stimuli

A predisposing factor to UTI is the postpartum period is

a. epidural anesthesia
b. manual removal of the placenta
c. retained placental fragments
a. epidural anesthesia

Seizures in neonates are signs of malfunctioning

a. brain stem activity
b. higher order areas of the brain
c. neuronal system
c. neuronal system

A woman is receiving oxytocin to address PPH. Dosages at a concentration above 30-40 u/L of IV fluids can lead to

a. fluid overload
b. hypertension
c. seizures
a. fluid overload

Determination of blood loss is difficult to assess in the postpartum period because of what physiologic change?

a. large loss of RBCs
b. persistence of high plasma volume
c. reversal of extracellular fluid accumulation
c. reversal of extracellular fluid accumulation

Subconjunctival hemorrhage is noted in a 2 hour old baby. The most likely cause is

a. early sign of conjunctivitis
b. irritation from the eye prophylaxis
c. ocular pressure experienced at birth
c. ocular pressure experienced at birth

Abnormal spinal nerve innervation should be suspected if there is

a. an absent babinski reflex
b. incomplete moro reflex
c. no tonic neck response
a. an absent babinski reflex

An obstetric factor that predisposes to uterine prolapse is

a. cervical cerclage
b. cesarean
c. vacuum delivery
c. vacuum delivery

A postpartum woman is recovering from placental abruption and PPH. She is diagnosed with DIC. The priority approach to care is

a. determination of underlying cause
b. immediate transfer to the ICU
c. replacement of blood
c. replacement of blood

The hat on the infant’s head prevents what type of heat loss?

a. convection
b. evaporation
c. radiation
a. convection

A priority of care prior to administering hemabate for PPH is to check if the patient has a history of

a. anemia
b. asthma
c. diabetes mellitus
b. asthma

A woman is receiving increasing pain meds for vaginal pain after a prolonged labor. Visual inspection for perineum looks normal with no vaginal bleeding. This patient needs to be further evaluated for

a. vaginal hematoma
b. retained placental fragments
c. undetected laceration
a. vaginal hematoma

Neonatal abstinence symptoms can also occur in the newborn secondary to maternal use of

a. antidepressants
b. dopaminergics
c. synthroid
a. antidepressants

Low set hairline, increased quantity of hair and brittleness found on examination of a neonate’s scalp is most likely d/t

a. congenital or genetic anomaly
b. ethnic, racial, or familial differences
c. normal variations seen in the general population
c. normal variations seen in the general population

A mother who delivered a baby with a cleft lip and palate is crying and does not want to see the baby. This is most likely reflective of feelings of the loss of the perfect child and

a. anger
b. entitlement
c. guilt
c. guilt

Respiratory acidosis occurs as a direct result of

a. asphyxia
b. hypoventilation
c. hypoxia
c. hypoxia

A couple who has experienced a perinatal loss from multiple congenital defects declines to see and hold the baby. The RN should

a. explain that many parents regret their choice to not see the baby later and they should reconsider
b. respect their wishes but know they may change their minds later
c. take a photo of the baby and give it to them at discharge
b. respect their wishes but know they may change their minds later

A breastfeeding woman developed endometritis 3 days postpartum following a cesarean. She was treated with IV antibiotic and discharged home on oral antibiotics. On her 6th postpartum day, she returns to the hospital with suprapubic pain, urinary frequency, urgency, and dysuria. There is blood in the urine. This is consistent with

a. cystitis
b. pelvic inflammatory disease
c. pyelonephritis
a. cystitis

A reversal agent for heparin is

a. protamine sulfate
b. prothrombin complex concentrates
c. vitamin K
a. protamine sulfate

Polycythemia occurs more frequently in the IUGR neonate whose mother has

a. an autoimmune disorder
b. diabetes
c. genetic thrombophilia
b. diabetes

Tachypnea, tremors, apnea, and hypothermia in a 12 hour old late term hypotonic neonate, born to a mother with DM would most likely be consistent with the onset of

a. congestive heart failure
b. hypoglycemia
c. sepsis
b. hypoglycemia

A breastfeeding woman’s migraine headaches are well controlled with ergotamine. She asks if she can continue while breastfeeding. She should be advised that it’s use in breastfeeding is generally

a. considered incompatible
b. safe to use
c. used if therapy is not prolonged
a. considered incompatible

Examination of the umbilical cord finds large amounts of clear drainage. This is consistent with

a. normal variant
b. localized infection
c. patent urachus
c. patent urachus

Failure to clear fetal lung fluid during delivery without labor usually results from

a. a complication of polyhydramnios
b. an esophageal anomaly
c. lack of catecholamine serge
c. lack of catecholamine serge

On the gestational age assessment, the examiner passes the arm over the other side of the body. This is

a. arm recoil
b. scarf sign
c. square window
b. scarf sign

Research has shown that postpartum exercise can

a. exacerbate fatigue in most woman
b. improve muscle tone and strength
c. lead to a faster uterine involution
c. lead to a faster uterine involution

Increased cardiac output in the postpartum period is most likely d/t increased

a. aortic pressure
b. heart rate
c. stroke volume
c. stroke volume

A trigger of the breast milk ejection reflux is a function of

a. high circulating prolactin levels
b. hypothalamic induced alveoli contraction
c. suckling-induced oxytocin release
c. suckling-induced oxytocin release

Treatment of a strawberry hemangioma includes

a. laser therapy for permanent resolution
b. no treatment unless it affects a vital function
c. early treatment because of it’s proliferative properties
b. no treatment unless it affects a vital function

In healthcare, providing info on the benefits as well as risks of an intervention is rooted in the ethical principle of

a. autonomy
b. beneficence
c. justice
a. autonomy

A factor that may require a neonate to receive an additional dose of vitamin K in the first week of life is

a. history of maternal 3rd trimester bleeding
b. maternal anticoagulation treatment during pregnancy
c. presence of prolonged prothrombin times
b. maternal anticoagulation treatment during pregnancy

Late preterm neonates are often poor feeders because of immaturity of

a. GI function
b. the coordination for suck, swallow, and breathing function
c. the respiratory control center in the brain
b. the coordination for suck, swallow, and breathing function

Treatment of a plugged duct in the breastfeeding woman will focus on

a. drainage of the duct
b. needle aspiration of the plug
c. vasodilation through heat and massage
c. vasodilation through heat and massage

Urinary retention is a common problem in the postpartum period d/t

a. decreased bladder capacity
b. dilation of ureters and renal pelvis
c. reduced sensation of intravascular fluid pressure
c. reduced sensation of intravascular fluid pressure

At discharge, a mother asks how she can best determine if her infant is growing appropriately. The best way to track growth is by the infant’s

a. head circumference
b. length
c. weight
c. weight

Women who are at higher risk for postpartum psychosis are those who have a history of

a. bipolar disorder
b. lack of social support
c. long time use of antidepressants
a. bipolar disorder

With antibiotic treatment, symptoms of mastitis usually resolve within

a. 1-2 days
b. 7 days
c. 10 days
c. 10 days

A neonate appears pink at rest but becomes deep red to purplish when crying. This is consistent with

a. choanal atresia
b. immature blood circulation
c. polycythemia
b. immature blood circulation

Non-shivering thermogenesis is the major way infants

a. contain heat
b. lose heat
c. produce heat
c. produce heat

After birth, a woman tells the RN that “nobody told me it would be like this.” The RN should recognize that

a. postpartum depression should be done
b. she may benefit from a psycho consult before discharge
c. this is a common feeling for new mothers
c. this is a common feeling for new mothers

Decreased blood pressure in the early postpartum period reflects

a. decreased plasma volume
b. orthostatic changes
c. rapid decline is cardiac output
c. rapid decline is cardiac output

The accuracy of a PKU screening depends on

a. adequate milk intake prior to the test
b. testing being done by 48 hours of life
c. testing being performed at 40 weeks gestation or later
a. adequate milk intake prior to the test

Pseudomenstruation can occur in the female neonate d/t

a. development of urate deposits
b. normal high specific gravity of neonatal urine
c. withdrawal of maternal hormones
c. withdrawal of maternal hormones

Preparation for the birth of a baby by the pregnant woman is integral to

a. maternal role transition
b. pregnancy validation
c. viewing the fetus as a separate individual
a. maternal role transition

A postpartum woman with preeclampsia remains of mag after delivery. It is important for the RN to monitor

a. blood pressure
b. cardiac activity
c. urine output
a. blood pressure

A predisposing factor to placenta previa is

a. adolescent pregnancy
b. high parity
c. twins
b. high parity

An antibody screen on an Rh negative mother is negative. The baby is Rh positive. This requires that Rh immunoglobulin be administered

a. at her 1st postpartum visit
b. based on the infant’s Coombs test results
c. within 72 hours postpartum
c. within 72 hours postpartum

When a woman has repeated bouts of plugged ducts, the most likely cause might be poor flow of milk d/t

a. fatigue
b. inadequate calories
c. inadequate fluids
c. inadequate fluids

Oxytocin is ordered for a woman who has steady vaginal bleeding despite fundal massage. This is done to

a. increase uterine muscle strength
b. promote coagulation factors
c. stimulate uterine contractions
c. stimulate uterine contractions

A woman with HELLP syndrome delivers via cesarean. She is stable and admitted to the postpartum unit. She is at increased risk for

a. adult RDS
b. generalized infection
c. pulmonary embolism
b. generalized infection

A breastfeeding woman with DM wants to go back to her oral hypoglycemia agent she was taking prior to her pregnancy. She should be advised that

a. drug transfer to breast milk is low so she can resume therapy
b. safety is unknown so the treatment is not recommended
c. she should remain on insulin until breastfeeding is well established
a. drug transfer to breast milk is low so she can resume therapy

In terms of lactation physiology, breast engorgement is a problem related to milk

a. production
b. removal
c. storage
b. removal

The fetal shunt that provides the pathways where the blood flows directly to the vena cava bypassing the liver is the

a. ductus arteriosis
b. ductus venosus
c. foramen ovale
b. ductus venosus

When anti-insulin hormones decrease in the postpartum insulin dependent woman, this causes

a. a lesser need for insulin
b. rebound insulinemia requiring increased glucose intake
c. sudden onset of hypoglycemia
a. a lesser need for insulin

A 42 year old woman who was on extended bedrest delivers via cesarean. Recovery has been normal but the patient is hypotensive and has chills. She has a history of asthma and hypertension. She now reports that she is feeling panicky because she is having difficulty breathing with some chest pain. Immediate attention is needed as these are signs of

a. acute asthma attack
b. pulmonary embolism
c. septic shock
b. pulmonary embolism

Cytomegalovirus is most commonly diagnosed in the neonate by viral isolation of

a. blood
b. nasopharyngeal aspirate
c. urine
c. urine

A 1 hour old infant is experiencing episodes of coughing and choking. This baby requires

a. immediate emergent surgery
b. intubation and ventilation
c. placement of an orogastric tube
c. placement of an orogastric tube

Maternal sulfonamides therapy in the last weeks of pregnancy can predispose the baby to

a. anemia
b. infection
c. jaundice
c. jaundice

A 2800 gram neonate, when lying on his side, shows a sharply demarcated red color seen in the dependent side of the body. The superior half is pale. If the infant is rotated to the other side, the color reverses. The appropriate intervention is to

a. assess temperature
b. continue to observe
c. obtain a hematocrit level
b. continue to observe

Maternal death or morbidity related to blood loss most commonly occurs d/t

a. catastrophic bleeding episode
b. ineffective management of slow, steady blood loss
c. late postpartum hemorrhage
c. late postpartum hemorrhage

When interpreting bilirubin levels, an essential factor for management is

a. age of infant
b. gestational age
c. maternal history
a. age of infant

A predisposing factor for periodic breathing is

a. prematurity
b. resuscitation at birth
c. traumatic delivery
a. prematurity

Endometritis is treated

a. based on blood culture results
b. based on endometrial culture results
c. with a broad spectrum antibiotic
c. with a broad spectrum antibiotic

A woman has been successfully breastfeeding. She has now developed unilateral mastitis. If no abscess is present, breastfeeding in this situation should

a. continue as prior to the infection
b. continue only if she is on antibiotics
c. should be done from the non-affected breast
a. continue as prior to the infection

A woman following cesarean is given Fentanyl for pain relief. To assess for potential side effects, it is important to monitor

a. blood pressure
b. respiratory rate
c. urine output
b. respiratory rate

A woman 48 hours post cesarean develops a prolonged spiking fever that is not responsive to antibiotic treatment. The most likely cause for these findings is

a. asymptomatic bacteruria
b. necrotizing fascitis
c. thrombophlebitis
c. thrombophlebitis

The resultant hypoxia from amniotic fluid embolism ultimately leads to a high rate of morbidity and mortality secondary to

a. cerebral encephalopathy
b. left-sided heart failure
c. massive hemorrhage
b. left-sided heart failure

The major effect of a breastfeeding woman engaging in breastfeeding 8-12 times a day is

a. enhancement of infant latch on and suck capabilities
b. maintenance of ongoing milk production
c. prolongation of antibody protection for the baby
b. maintenance of ongoing milk production

Following a prolonged labor and delivery, a baby has an elongated head with bruising on the back of the head. The crown of the head is spongy to the touch and edema is widespread and not confined. This condition

a. requires a full neurologic assessment
b. requires an x-ray evaluation
c. should resolve on it’s own in a few days
c. should resolve on it’s own in a few days

The most common infection in the postpartum period is

a. cystitis
b. endometritis
c. mastitis
b. endometritis

A woman diagnosed with preeclampsia delivered 8 hours ago and was taken of mag sulfate 3 hours ago. She now develops severe upper abdominal pain with nausea and vomiting. This could be

a. impending eclamptic seizures
b pulmonary embolism
c. side effects from mag treatment
a. impending eclamptic seizures

Malrotation with midgut volvulus (twisted intestines) requires emergent surgery to prevent

a. intussusception
b. irreversible intestinal infarction
c. peritonitis
b. irreversible intestinal infarction

A breastfeeding woman who has been breastfeeding for one week reports nipple pain. The most likely cause is

a. poor breast hygiene
b. possible thrush infection
c. too frequent feedings
b. possible thrush infection

A woman is admitted to the postpartum unit following delivery en route to the hospital. The RN notes fresh bruising on the back and shoulders and asks if anyone is hurting her. She says no and states that she fell down the stairs yesterday. The RN should

a. accept her explanation
b. ask her the question later on
c. refer her to the social worker to follow up
b. ask her the question later on

The sequelae of catecholamine release, mild acidosis, and cold stress manifests in the neonate in the first few hours of life as

a. acrocyanosis
b. jaundice
c. petechiae
a. acrocyanosis

A distinguishing feature of hydrocele that would differentiate it from other GU problems is

a. ability for transillumination
b. presence of pain on palpation
c. scrotal color
a. ability for transillumination

The fall in pulmonary vascular resistance at birth results in closure of the

a. ductus arteriosis
b. ductus venosus
c. foramen ovale
a. ductus arteriosis

On uterine assessment, the RN notes the fundus is not firm and initiates gentle massage with the dominant hand and supports the lower segment with the non-dominant hand in order to

a. assess for bladder distention
b. evaluate for any pooling of blood
c. prevent uterine eversion
c. prevent uterine eversion

Decreased GI motility and muscle tone in the postpartum woman can lead to

a. exacerbated afterpains
b. gaseous distention
c. heart burn
b. gaseous distention

A predisposing factor to DVT is varicosities secondary to

a. altered coagulation
b. venous stasis
c. venous wall damage
b. venous stasis

A late preterm infant who is experiencing difficulty in breastfeeding is at increased risk to develop hyperbilirubinemia because of

a. inadequate RBD turnover
b. increased enterohepatic circulation of bilirubin
c. overactive glucuronidation
b. increased enterohepatic circulation of bilirubin

After a long labor and cesarean, the mother gives the baby to her husband. The RN should recognize that this woman’s lack of interest is most likely d/t

a. early onset of baby blues
b. fatigue and pain
c. inexperience as a mother
b. fatigue and pain

When lambdoid sutures are overriding with the parietal bone on top of the occipital bone, this is consistent with

a. craniotabes
b. molding
c. subgaleal hemorrhage
b. molding

When there is a narrowing of the upper thoracic aorta that produces an obstruction, this is manifested as what heart condition?

a. coarctation of the aorta
b. Marfan’s syndrome
c. tetralogy of fallot
a. coarctation of the aorta

Movement of the milk from the ducts to the nipple is initiated by suckling which stimulates

a. lactogenesis
b. release of prolactin
c. the let down reflex
c. the let down reflex

Neonatal abstinence scoring system includes the observation of various symptoms. One of them is

a. lethargy
b. low urine output
c. unstable temperature
c. unstable temperature

There is great variation in the return of menses in the breastfeeding woman because of

a. depressed estrogen levels
b. fluctuation in prolactin levels
c. higher pituitary sensitivity to luteinizing hormone (LH)
a. depressed estrogen levels

The RN would anticipate that a term newborn with a CV condition where coronary blood flow occurs in a retrograde manner into a small ascending aorta through the patent ductus arteriosis would present with

a. cyanosis
b. no murmur
c. severe pallor and grayish skin tone
c. severe pallor and grayish skin tone

The type of umbilical cord care that has demonstrated the best way to prevent infection is

a. antimicrobial ointment after bathing
b. rapid drying and keeping the cord clean
c. alcohol or triple dye
b. rapid drying and keeping the cord clean

A breastfeeding woman with DM type 2 should be encouraged to include additional snack during the day to

a. blunt postprandial glucose spikes
b. improve insulin utilization
c. prevent hypoglycemia
c. prevent hypoglycemia

oligohydramnios in pregnancy predisposes the fetus to

a. amniotic band syndrome
b. fetal adhesions
c. urinary tract obstructions
a. amniotic band syndrome

The underlying problem with a plugged duct in breastfeeding is

a. anatomic variant
b. inflammation
c. milk stasis
c. milk stasis

Irritability in the breastfeeding infant can be caused by exposure to

a. environmental allergens
b. high maternal intake of caffeine
c. nicotine from maternal smoking
c. nicotine from maternal smoking

The results of a L/S assay shows a ratio of 1:1 at 35 weeks gestation. This finding

a. is a sign of lung immaturity
b. is normal for gestational age
c. requires further evaluation for PG
b. is normal for gestational age

A positive Ortolani maneuver indicates

a. brachial plexus injury
b. dysplasia of the hip
c. positional club foot
b. dysplasia of the hip

The factor that most predisposes an infant to vitamin K deficiency is

a. absence of intestinal flora
b hepatic immaturity
c. reduced liver stores
a. absence of intestinal flora

A woman who was treated for DVT during pregnancy will continue anticoagulation in the postpartum period. The order is to add Coumadin to the Heparin treatment. The woman is breastfeeding. The RN would

a. add the Coumadin to the treatment
b. double check that both are needed
c. question the safety of the order
c. question the safety of the order

As the transition of colostrum to mature milk occurs, there is an increase in what component?

a. CHO
b. fat
c. protein
b. fat

A woman who delivered via cesarean 10 days ago is being visited by the home health RN. The woman is pale, tachycardic, and hypotensive. She states she has some profuse vaginal bleeding earlier in the day and she is feeling weak and dizzy. Subinvolution is evident on exam. This scenario is most consistent with

a. coagulation problems
b. endometritis
c. retained placental fragments
c. retained placental fragments

In examining a neonate with Erb’s palsy, it would be expected to paralysis of the

a. hand and wrist
b. neck and diaphragm
c. upper arm and shoulder
c. upper arm and shoulder

According to AWHONN, the RN patient ratio for normal, healthy couplets should be no more than

a. 1:2
b. 1:3
c. 1:4
c. 1:4

Baby James, born by elective cesarean to a 25 year old mother, weighs 3265 grams/7.3 lbs. RN Lucia places the baby under the warmer. In addition to the assessment, the RN should closely monitor

a. temperature instability d/t type of birth
b. respiratory distress d/t lack of contractions
c. signs of acrocyanosis
d. unstable blood sugars
b. respiratory distress d/t lack of contractions

Baby Nikki, born 2 hours ago has just arrived in the newborn nursery. Which nursing measure will prevent the baby from losing heat due to evaporation?

a. keeping him away from drafts
b. putting a blanket between him and cold surfaces
c. putting a cap on his head
d. drying him thoroughly after birth
d. drying him thoroughly after birth

The baby of a mother with DM type I is at high risk for hypoglycemia. An initial sign an RN should recognize as indicating hypoglycemia is

a. peripheral acrocyanosis
b. bradycardia
c. lethargy
d. jaundice
c. lethargy

Baby Bonnie is admitted to the nursery following a long and difficult labor. Admission VS are temp 96.5, HR 168, and RR 64. After placing the baby under the warmer, the RN’s next action should be

a. performing a full assessment
b. checking the newborn’s glucose level
c. reviewing the pregnancy and delivery history
d. calling the pedi to report findings
b. checking the newborn’s glucose level

During the admission assessment of a newborn female, the RN notes a large lump on the baby’s head. Concerned about making the correct assessment, the RN differentiates between caput succedaneum and a cephalohematoma based on the knowledge that

a. a cephalohematoma doesn’t cross suture lines
b. caput succedaneum occurs primarily with primigravidas
c. a cephalohematoma occurs with a birth that required instruments
d. caput succedaneum occurs primarily with a prolonged 2nd stage of labor
a. a cephalohematoma doesn’t cross suture lines

Nurse Agnes is assessing a neonate born 1 day ago to a mother who smoked one pack of cigarettes during pregnancy. Which finding is most common in neonates whose mothers smoked during pregnancy?

a. post-term birth
b. LGA
c. SGA
d. AGA
c. SGA

Nurse Heidi is aware that the following is not a contributing factor to unstable blood sugars in the newborn?

a. prematurity
b. respiratory distress
c. postdated infant
d. cesarean delivery
d. cesarean delivery

While caring for a healthy baby girl, the RN notices red stains on the diaper after the baby voids. Which of the following should the RN do next?

a. call the MD to report findings
b. encourage the mother to feed the baby to decrease dehydration
c. do nothing because it’s normal
d. check the baby’s urine for hematuria
c. do nothing because it’s normal

Baby Joy, a full-term neonate is diagnosed with hydrocephalus. Assessment is most likely to reveal

a. wide or bulging fontanels
b. a decreased occipitofrontal circumference
c. upward-slanting eyes
d. heightened alertness
a. wide or bulging fontanels

Six hours after birth, a baby is transferred to the nursery to prevent hypothermia. What is a common source of radiant heat loss?

a. low room humidity
b. cool weight scale
c. cool incubator walls
d. cool room temp
c. cool incubator walls

A baby begins to gag and turns a dusky color. What should the RN do first?

a. calm the baby
b. notify the MD
c. provide O2 via face mask as ordered
d. aspirate nose and mouth with a bulb syringe
d. aspirate nose and mouth with a bulb syringe

A mother plans to bottle feed her full term baby. What is normal during the first 24 hours?

a. 120mL of formula q 2-3 hours
b. 80-100mL of formula q 2 hours
c. 40-46mL of formula q 2-4 hours
d. 20-40mL of formula q 3 hours
d. 20-40mL of formula q 3 hours

Celia is concerned because her breastfeeding infant wants to nurse all the time. Which of the following responses is most appropriate?

a. breast milk is ideal for the baby, so his stomach will digest it quickly, requiring more feedings
b. let me call the LC to make sure that your baby is feeding properly
c. don’t worry, your baby is an aggressive feeder and needs a lot of sucking satisfaction
d. it seems as if your baby is hungry, why don’t you provide your baby with formula after feedings to make sure he is getting enough nourishment
a. breast milk is ideal for the baby, so his stomach will digest it quickly, requiring more feedings

A baby must receive an eye preparation to prevent opthalmia neonatorum. How should the RN administer this medication?

a. by avoiding holding the eyelid open during administration
b. by letting the medication drip onto the surface of the eye
c. by positioning the baby so the head remains still
d. by holding the baby in the football position
c. by positioning the baby so the head remains still

A patient who admits to using heroin delivers at 32 week infant. Who neonatal assessment is most important for the RN to perform?

a. auscultation of breath sounds for pulmonary problems
b. careful observation of respiratory effort because of the prematurity
c. evaluation for signs of withdrawal
d. observation for jaundice
c. evaluation for signs of withdrawal

A mother just gave birth to twins. Twin “A” weighs 2500g/5.8lb. and Twin “B” weighs only 1900g/4.3lb. In addition to routine care, the MD ordered that Twin “B” be kept in an isolette to help maintain his temperature. Based on the latest research, the RN might suggest which of the following interventions in place of using an isolette to maintain the temperature?

a. increasing the number of calories to support a stable temp
b. wrapping the baby in 2 blankets with a hat and monitor for low temps
c. placing the twins in the same crib so the larger baby can keep the smaller baby warm
d. placing a hot-water bottle in the crib of Twin “B”
b. wrapping the baby in 2 blankets with a hat and monitor for low temps

A patient plans to breastfeed her healthy, full-term baby. The RN encourages her to start breastfeeding within 30 minutes of birth because

a. the neonate will be responsive and eager to suck at this time
b. breastfeeding will inhibit prolactin production
c. her breasts will be firm and filled with colostrum at this time
d. breastfeeding will help the baby fall asleep
a. the neonate will be responsive and eager to suck at this time

The RN is teaching the mother of an infant about the importance of immunizations. The RN should teach her that active immunity

a. develops rapidly and is temporary
b. occurs by antibody transmission
c. results from exposure of an antigen through immunizations or direct contact
d. may be transferred by mom to baby
c. results from exposure of an antigen through immunizations or direct contact

The RN is aware that the following caloric intake is normal for a newborn

a. 110-130/kg
b. 30-40/lb
c. at least 2mL per feeding
d. 90-100/kg
a. 110-130/kg

A patient gives birth to a baby prematurely at 28 weeks. To obtain the Apgar score, the RN assesses the neonates

a. temperature
b. respirations
c. blood pressure
d. weight
b. respirations

How does the RN assess the rooting reflex on a newborn?

a. placing an object in the neonate’s palm
b. stroking the sole of the foot
c. stroking the cheek
d. touching the lips
c. stroking the cheek

The RN is caring for a baby with a myelomeningocele. The priority nursing intervention is

a. ensuring adequate nutrition
b. preventing infection
c. promoting neural tube defect
d. conserving body heat
b. preventing infection

During neonatal resuscitation immediately after delivery, chest compressions should be initiated when the heart rate falls below

a. 60/min
b. 80/min
c. 100/min
d. 110/min
a. 60/min

During a bath, a female baby has a nursing diagnosis of “Risk for injury related to slippage while bathing.” Which interventions best addresses the diagnosis?

a. hold the baby loosely and gently
b. support the baby’s head and back with the forearm
c. use one hand to support the baby’s head
d. strap the baby into the bath basin
b. support the baby’s head and back with the forearm

A pregnant client at risk for the development of pregnancy-induced hypertension is one with a history of

a. AIDS
b. anemia
c. oligohydramnios
d. renal disease
d. renal disease

A non-stress test is performed on a mother who is 35 weeks gestation who also has DM. Within a 20 minute period, the fetal HR increases from 120-140/min on 2 occasions, lasting 15 seconds each. The test should be interpreted as

a. nonreactive
b. positive
c. reactive
d. unreliable
c. reactive

A baby with meconium stained fluid is suctioned when the head is delivered. Poor muscle tone and HR of 80/min are noted. The next step in the resuscitation process is

a. begin positive pressure ventilation
b. dry and stimulate the newborn
c. evaluate respiratory efforts
d. suction the trachea with an endotracheal tube
b. dry and stimulate the newborn

Immediately after delivery, the RN notes that the baby has a scaphoid abdomen and a right shift cardiac point of maximum intensity. The condition that is most consistent with these signs is

a. abdominal wall defect
b. diaphragmatic hernia
c. duodenal atresia
d. tracheoesophageal fistula
b. diaphragmatic hernia

During an initial newborn assessment, the finding that needs further eval is

a. excessive mucous
b. irregular respiratory rate of 60/min
c. lack of audible bowel sounds
d. slight rales on auscultation
d. slight rales on auscultation

Erb’s palsy is suspected in a large baby with shoulder dystocia. Findings most consistent with this type of palsy include

a. adduction, pronation, internal rotation of the arm
b. fractured clavicle
c. lack of movement in lower extremities
d. loss of movement and hand paralysis
b. fractured clavicle

An example of an abnormal finding during a baby assessment is

a. negative Barlow maneuver
b. periodic breathing
c. bounding pulses
d. presence of “witch’s milk”
c. bounding pulses

A contraindication of breastfeeding is

a. active genital herpes
b. maternal antibiotic treatment with Claforan (cefotaxime)
c. silicone breast implants
d. untreated maternal TB
d. untreated maternal TB

The best description of the breastfed baby’s stool is

a. frequent and loose
b. pasty, greenish brown large curds
c. liquid with small yellow curds
d. infrequent and watery
b. pasty, greenish brown large curds

The MD orders urine for reducing substance. This test is ordered to assess for

a. phenylketonuria
b. galactosemia
c. cystic fibrosis
d. maple syrup urine disease
b. galactosemia

A neonate is circumcised using a plastibell. Because the plastibell was used

a. tub baths can be used
b. petroleum gauze should be avoided
c. healing time is reduced
d. soap and water should be used with each diaper change
b. petroleum gauze should be avoided

The current guidelines for a baby delivered to a hepatitis B positive mother include

a. hepatitis B vaccination at 48 hours of life
b. delayed breastfeeding until after the newborn is immunized
c. HBIG any time before discharge
d. HBV and HBIG within 12 hours of life
d. HBV and HBIG within 12 hours of life

Random tests of the newborn for maternal drug abuse without clinical suspicion are

a. performed as part of routine admission
b. illegal if no parental consent
c. ethical in most situations
d. indicated in all high-risk populations
d. indicated in all high-risk populations

A nonhemolytic cause of neonatal jaundice includes

a. twin-to-twin transfusion
b. ABO incompatibility
c. spherocytosis
d. G6PD deficiency
d. G6PD deficiency

The mother of a baby who is SGA has a history of heavy smoking throughout the pregnancy. The RN should further evaluate the baby’s

a. potassium levels
b. blood gas levels
c. hematocrit
d. blood type and Rh factor
b. blood gas levels

The infant of a diabetic mother is showing signs of hypoglycemia within 30 minutes of delivery. The baby’s RR is 68/min. The RN should anticipate an order for

a. immediate oral formula feeding
b. breastfeeding ASAP
c. gavage feeding with 10% glucose
d. placement of an IV line
b. breastfeeding ASAP

SGA babies are at greatest risk for

a. anemia
b. hyperthermia
c. meconium aspiration
d. hyperglycemia
c. meconium aspiration

A 1 day old, term baby has difficulty feeding, accompanied by vomiting and diarrhea. The baby becomes irritable with a high-pitched cry. These findings are most consistent with

a. drug withdrawal
b. intestinal obstruction
c. hypoglycemia
d. intraventricular hemorrhage
a. drug withdrawal

A quantitative rapid plasma regain test on a newborn is positive. The RN should document any other signs of suspected

a. gonorrhea
b. chlamydia
c. herpes
d. syphilis
d. syphilis

The drug choice in a baby with herpes infection is

a. PCN
b. acyclovir
c. erythromycin
d. zidovudine
b. acyclovir

In planning the delivery of a 42 week old pregnancy, the RN should observe the baby for

a. RDS
b. meconium aspiration
c. anemia
d. transient tachypnea of the newborn
b. meconium aspiration

A pregnancy history reveals the presence of polyhydramnios. The baby should be assessed for an abnormality of the

a. GI system
b. urinary system
c. pulmonary system
d. reproductive system
b. urinary system

A triple prenatal test is very sensitive in identifying newborns with

a. IUGR
b. trisomy 13
c. open neural tube defects
d. down syndrome
d. down syndrome

A newborn has the following characteristics: SGA, hepatosplenomegaly, and petechiae. These signs are consistent with a diagnosis of TORCH. A disease in this group would be

a. syphilis
b. CMV
c. thrombocytosis
d. rubeola
b. CMV

A 2 week postpartum mother c/o loss of appetite, difficulty concentrating, and sleeplessness. These symptoms are consistent with postpartum

a. psychosis
b. OCD
c. depression
d. panic disorder
c. depression

A 35 week pregnant patient developed GDM. Due to this, the MD ordered a lab test to evaluate the status of the fetus. A noninvasive fetal assessment test appropriate for the specific gestation would be

a. CST – contraction stress test
b. NST – non-stress test
c. amniocentesis
d. scalp pH
b. NST – non-stress test

The highest possible score for a biophysical profile indicating a healthy baby is

a. 4
b. 6
c. 8
d. 10
d. 10

A patient in labor has a prolonged second stage. The neonatal sign most likely related to this situation is

a. facial nerve damage
b. hypoglycemia
c. retained lung fluid
d. congenital hip dislocation
c. retained lung fluid

Following epidural anesthesia, a labor patient experienced hypotension. After delivery, the baby must be evaluated for

a. tachycardia
b. hyperactivity
c. asphyxia
d. tachypnea
c. asphyxia

Of the following factors, the one that indicates the need to plan for potential neonatal resuscitation is

a. maternal age 30
b. early decels
c. 38 week gestation
d. hydramnios
d. hydramnios

The RN is preparing equipment for neonatal resuscitation. Equipment for airway management includes

a. straight laryngoscope blade
b. 5.0mm endotracheal tubing
c. resuscitation bag with pressure release valve
d. noncushioned mask
c. resuscitation bag with pressure release valve

An accurate decision in managing a baby’s airway during resuscitation would be

a. suction the nose and then the mouth
b. use PPV for HR of 130/min
c. use PPV for gasping respirations
d. administer free-flow O2 for apnea
c. use PPV for gasping respirations

During neonatal resuscitation, the step that follows implementation of warmth and suctioning is

a. drying the neonate
b. assigning an Apgar score
c. providing bag and mask ventilation
d. determining the HR
c. providing bag and mask ventilation

A baby has been resuscitated with a least 30 seconds of PPV with 100% O2. The baby’s HR remains 40/min despite chest compressions. The drug indicated in this situation is

a. sodium bicarbonate
b. naloxone hydrochloride
c. a volume expander
d. epinephrine
d. epinephrine

The preferred route of administration for naloxone hydrochloride is

a. IM
b. oral
c. endotracheal
d. SubQ
a. IM

After delivery, a baby is pink when crying and cyanotic when quiet. Respirations appear noisy. These signs are consistent with

a. diaphragmatic hernia
b. choanal atresia
c. infection
d. pneumothorax
b. choanal atresia

The RN suspects that a neonate has a diaphragmatic hernia. The RN should anticipate that the next intervention is to

a. insert an endotracheal tube
b. use bag/mask ventilation
c. insert an orogastric tube
d. assist with the insertion of an umbilical line
c. insert an orogastric tube

A NP needs to treat tension pneumothorax in a neonate. The RN assisting with the procedure must immediately prepare

a. a bag and mask
b. 100% O2 by hood
c. a chest tube insertion and suctions
d. a butterfly needle, stopcock, and syringe
b. 100% O2 by hood

Emergency drugs must be administered to a 28 week, low birth weight neonate. The RN should anticipate insertion of

a. umbilical artery cath
b. umbilical venous cath
c. peripheral line
d. percutaneous venous line
b. umbilical venous cath

During the 2nd stage after birth, a baby should exhibit

a. sleep
b. tachycardia
c. vomiting
d. excessive oral mucous
a. sleep

Care of a normal, term baby in transition includes

a. checking the rectal temp q 30-36 minutes
b. instilling prophylactic eye medication within one hour of delivery
c. providing respiratory support if the baby’s RDS score is 0
d. administer vitamin K at 2 hours of life
d. administer vitamin K at 2 hours of life

When noted during initial neonatal assessment, the sign that requires further evaluation is

a. capillary refill of 3 seconds
b. apnea of 40 seconds
c. spontaneous jerks and twitches
d. spitting of clear mucous
b. apnea of 40 seconds

Persistent pulmonary hypertension in a newborn is associated with

a. large babies
b. hypocalcemia
c. tachycardia
d. hypoxia
d. hypoxia

A baby is born at 37 weeks gestation by cesarean. One hour after birth, the baby develops mild tachypnea, mild retractions, and nasal flaring. Minimal central cyanosis is also noted. Transient tachypnea of the newborn (retained lung fluid) is suspected. The expected treatment would be

a. isolation
b. antibiotics
c. supplemental O2
d. chest percussion
c. supplemental O2

A newborn has asymmetric Moro reflex. The left side is normal and the reflex is absent on the right side. These findings are most likely due to

a. Klimpke palsy
b. fractured clavicle
c. Horner syndrome
d. temporary sensory deficit
b. fractured clavicle

A neonate who had a traumatic breech delivery is discharged from the hospital. Within 3 days, the infant is readmitted with RDS. The RN should suspect injury to

a. the brachial plexus
b. 7th cranial nerve
c. C8 to T1
d. the phrenic nerve
a. the brachial plexus

Two hours after a vacuum extraction, a newborn demonstrates pallor and hypotonicity. The scalp is tight and boggy on palpation. A head scan is ordered because these signs are consistent with

a. periventricular hemorrhage
b. depressed skull fracture
c. subgaleal hemorrhage
d. caput succedaneum
c. subgaleal hemorrhage

During assessment of an AGA baby, the RN notes clear, superficial, small vesicles on the face and scalp. No redness is noted. Milia is suspected. The RN should

a. request an order for a platelet count
b. check maternal records for a history of psoriasis
c. document the findings in the chart and move the newborn to a cooler environment
d. apply a bacterial antibiotic cream
c. document the findings in the chart and move the newborn to a cooler environment

Which of the following signs in a baby must be immediately evaluated by the RN?

a. RR of 40/min
b. heart gallop
c. pectus excavatum
d. chest circumference smaller than head circumference
b. heart gallop

A newborn has a flat affect, epicanthal folds, and abnormal ears. Down syndrome is suspected. Other findings that support this diagnosis include

a. deep blue sclerae
b. excessive tearing
c. hairy ears
d. brush spots
d. brush spots

During an assessment of a 1 day old, term baby, the finding that would be considered a normal variation that requires no specific intervention is

a. purulent umbilical cord drainage
b. presence of one vein and one artery
c. palpable liver edge below the ribs
d. flabby sunken abdomen
b. presence of one vein and one artery

A positive Ortolani maneuver and positive radiograph confirm a diagnosis of developmental dysplasia of the hip. In planning care, the RN expects the 1st treatment to be

a. pavlik harness
b. traction
c. closed reduction
d. spica cast
a. pavlik harness

A gestational assessment reveals the following: weight of 8lbs, flexed posture, square window of 0-degree angle, popliteal angle of less than 90 degrees and leathery cracked skin. The baby would most likely

a. have an extremely low birth weight
b. be preterm
c. be postterm
d. be 34 week gestation
c. be postterm

A HIV positive mother asks whether she can breastfeed. The response should be

a. no problem if the mother is receiving treatment for HIV
b. contraindicated
c. the mother’s choice
d. delayed until maternal treatment for HIV has begun
b. contraindicated

The most appropriate nursing intervention for breast engorgement is to

a. administer frequent narcotic pain medication
b. encourage manual expression after feedings
c. encourage 4 hour intervals between feedings
d. apply cold packs between feedings
b. encourage manual expression after feedings

To ensure the safety of stored milk, the mother should be instructed to

a. discard milk left at room temperature for more than 1 hour
b. warm milk in the microwave on a low setting
c. discard thawed milk refrigerated for 24 hours
d. refreeze any leftover thawed milk within 2 hours
c. discard thawed milk refrigerated for 24 hours

To minimize bacterial contamination of formula, parents should be taught that bottles must be

a. sterilized
b. refrigerated if not totally consumed
c. refrigerated for no longer than 7 days
d. used at one feeding
d. used at one feeding

A single S2 click is noted in a newborn. The difference in BP between the upper and lower extremities is more than 20mmHg. This finding is consistent with

a. normal variation
b. coarctation
c. congenital HTM
d. CHF
b. coarctation

A 1 day old, term newborn becomes jaundiced. Total bilirubin levels are 6mg at 8AM and 11mg at noon. The RN should anticipate further eval for

a. hepatitis A
b. hypoglycemia
c. hemolysis
d. dehydration
c. hemolysis

Normal finding of urinary elimination in the newborn is

a. failure to void within 49 hours
b. three total voiding by day 2
c. dark yellow color
d. urate crystals in the first 24 hours
d. urate crystals in the first 24 hours

The RN is obtaining a blood sample for a newborn genetic screening test. The sampling method with the highest probability for accuracy involves obtaining the sample

a. prior to 24 hours of age
b. from the first drop of blood
c. before antibiotic administration
d. from the cord blood
c. before antibiotic administration

An expected treatment for maple syrup urine disease is

a. limiting dietary leucine
b. restricting phenylalanine in diet
c. administering oral biotin
d. giving pancreatic enzymes
a. limiting dietary leucine

Parents request an explanation of the reason for their newborn’s diagnosis of PKU. The RN’s response is based on the parent’s genetic history and the fact that the diease is

a. autosomal dominant
b. autosomal recessive
c. a chromosomal defect
d. multifactoral
b. autosomal recessive

Which intervention is helpful when counseling parents of a newborn with a chromosome disorder?

a. delaying the discussion of the abnormality until the diagnosis is complete
b. providing an in-depth discussion using medical terms when first suspected
c. having the newborn with the parents when they are informed of the condition
d. showing the least possible emotional involvement
c. having the newborn with the parents when they are informed of the condition

Uncircumcised newborns are at risk for

a. genital ulcerative disease
b. chordee
c. clotting problems
d. UTIs
d. UTIs

After circumcision without anesthesia, a newborn is irritable, wake frequently, and takes less formula. These findings are most likely indicating

a. infection
b. pain
c. urologic complications
d. nerve injury
b. pain

A method of pain relief considered effective for a newborn undergoing circumcision is

a. local warm packs
b. use of anesthesia with epi
c. buffered lidocaine
d. topical anesthetic cream
d. topical anesthetic cream

The RN is assisting with a circumcision. Once the plastibell is fitted over the penis, the MD ties a suture around the rim, and cuts away the excess prepuce. The RN’s responsibility after the circumcision done with a plastibell includes

a. cleaning the circ site with soap and water
b. placing petroleum gauze around the area
c. loosening the tie if it’s too tight
d. documenting the procedure
d. documenting the procedure

For a circumcision completed with a Gomco clamp, the type of dressing indicated is

a. petroleum gauze
b. 4×4 gauze
c. lidocaine cream
d. antibiotic cream
a. petroleum gauze

The current recommendation for administering the hepatitis B vaccine to a baby is

a. at 1 week of age
b. prior to hospital discharge
c. at 1-2 months of age
d. at the first check up
b. prior to hospital discharge

If maternal hepatitis B is negative, the newborn should receive

a. HBIG
b. no hepatitis vaccine
c. routine antiviral drugs
d. hepatitis B vaccine prior to discharge
d. hepatitis B vaccine prior to discharge

An adverse effect associated with the administration of the rubeola vaccine is

a. meningitis
b. Guillain-Barre syndrome
c. SIDS
d. polyneuropathy
b. Guillain-Barre syndrome

A parent wants info regarding contraindications to immunizations. A definite contraindication would be

a. mild upper respiratory infection
b. encephalopathy within 15 days of prior dose
c. prematurity
d. low grade fever
b. encephalopathy within 15 days of prior dose

A hospital has a large population of newborns delivered to mothers suspected of drug abuse. Neonatal lab tests to screen for this problem must be

a. performed on all newborns
b. revealed to authorities before parents
c. used to direct immediate medical interventions
d. performed even if the parents disagree
c. used to direct immediate medical interventions

In adoption cases, what decision must be made according to individual state law rather than by the birth mother?

a. consent to screening procedures
b. visitation by the adoptive family
c. mother’s involvement in care
d. rights of the birth father
b. visitation by the adoptive family

Which situation involving a hospitalized baby necessitates a follow up referral to child protective services?

a. discharge against medical advice
b. parental refusal of hepatitis B vaccine
c. religious concern about PKU screening
d. parental request for copies of all hospital records
a. discharge against medical advice

In physiologic jaundice, the total serum bilirubin at 48 hours of age should not exceed

a. 5-6
b. 8-9
c. 10-11
d. 12-13
d. 12-13

The neonate at highest risk for developing early signs of hyperbilirubinemia is one

a. who has anemia
b. who has polycythemia
c. who is being formula fed
d. who is female
b. who has polycythemia

Appropriate treatment for a baby with breastfeeding associated jaundice includes

a. interrupting breastfeeding for 24 hours
b. using a pacifier
c. pumping after nursing
d. giving glucose water between feedings
c. pumping after nursing

A baby undergoing phototherapy develops bronze baby syndrome. This baby should be evaluated for

a. cholestatic jaundice
b. blood incompatibilities
c. milk allergies
d. lactose intolerance
a. cholestatic jaundice

A baby with jaundice has a total bilirubin of 12mg at 36 hours of life. An expected lab order would be

a. potassium level
b. liver scan
c. urine culture
d. CBC
d. CBC

The RN is checking the maternal history of a baby with polycythemia. A condition that supports the diagnosis is

a. premature delivery of the baby
b. maternal alcohol use
c. maternal diabetes
d. maternal hypertension
c. maternal diabetes

A baby has a venous hematocrit of 63% and is diagnosed with symptomatic polycythemia. The sign consistent with this diagnosis is

a. increased urine output
b. respiratory distress
c. hyperglycemia
d. hypercalcemia
b. respiratory distress

Anemia should be suspected in a newborn who

a. is postterm and SGA
b. has a diabetic mother
c. is the smaller twin
d. is a term baby with plethoric color
a. is postterm and SGA

A neonate is diagnosed with chronic blood loss during pregnancy. The factor that is significant in differentiating this diagnosis from acute blood loss is

a. tachycardia
b. low blood pressure
c. weak pulses
d. hepatomegaly
a. tachycardia

Evaluation of hypoglycemia should occur as soon as possible after birth for a baby with

a. macroglossia
b. Apgar score of 7
c. 41 weeks gestation
d. maternal history of PKU
c. 41 weeks gestation

A neonate is suspected of the development of hypoglycemia. This diagnosis is consistent with

a. temperature fluctuations
b. bradycardia
c. hypertonicity
d. jaundice
a. temperature fluctuations

What information is accurate when evaluating a newborn for hypoglycemia?

a. capillary samples are lower than venous samples
b. repeat screenings should be done every hour for 24 hours
c. high-risk newborns without signs should be evaluated at 4 hours of age
d. if signs are present, they should be evaluated immediately
d. if signs are present, they should be evaluated immediately

The best treatment for a non-responsive, lethargic baby diagnosed with symptomatic hypoglycemia is

a. IV glucose
b. breastfeeding
c. 50% glucose per gavage
d. nipple feeding of glucose water
a. IV glucose

A LGA baby of a diabetic mother should be specifically assessed for

a. anemia
b. cardiac anomalies
c. meconium aspiration
d. hypercalcemia
b. cardiac anomalies

In managing the infant of a diabetic mother, the RN should

a. observe for hypotension
b. expect signs of hyperglycemia
c. check for polycythemia
d. follow potassium levels closely
c. check for polycythemia

Typical complications of a 40 week, SGA baby include

a. hypocalcemia
b. decreased blood viscosity
c. hypometabolism
d. RDS
a. hypocalcemia

A SGA baby is at high risk for

a. hyperglycemia
b. low hematocrit
c. polycythemia
d. hypercalcemia
c. polycythemia

A labor patient is suspected of cocaine abuse. Her blood pressure is 170/100 and she has arrhythmia. The client is at risk for

a. polyneuritis
b. postmaturity
c. seizures
d. delirium
c. seizures

A newborn assessment reveals IUGR with microcephaly, thin upper lip, and flattened philtrum. The RN suspects maternal abuse of

a. cocaine
b. alcohol
c. amphetamines
d. heroin
b. alcohol

A newborn who is jittery is suspected of having neonatal abstinence syndrome. To rule out other possible causes of jitteriness, the RN would anticipate an order to check

a. hypocalcemia
b. hyperglycemia
c. hypermagnesemia
d. hypothyroidism
a. hypocalcemia

Breastfeeding is considered safe is mothers who take

a. acetaminophen
b. cocaine
c. alcohol
d. amphetamines
a. acetaminophen

An expected finding in the neonate of a mother with a history of marijuana abuse would be

a. minor anomalies
b. SGA
c. decreased visual reaction
d. unresponsive Moro reflex
b. SGA

Maternal history reveals abuse of cocaine. Expected nursing interventions specific to this drug include

a. assessment of stooling pattern
b. administration of phenobarbital
c. encouragement of the mother to breastfeed
d. follow up of maternal blood type
a. assessment of stooling pattern

A newborn is suspected of having a bacterial infection. Clinical manifestations consistent with this condition include

a. plethora and jitteriness
b. hypertonicity and excessive hunger
c. high levels of alertness and crying
d. apnea and diminished activity
c. high levels of alertness and crying

A work-up for possible infection is completed on a newborn. One finding that would be considered normal is

a. positive blood culture
b. WBC of 7000/mm
c. high C-reactive protein level
d. positive latex agglutination
c. high C-reactive protein level

A newborn has documented sepsis. Antibiotics should be administered for a minimum of

a. 3 days
b. 5-7 days
c. 10-14 days
d. 4 weeks
c. 10-14 days

An expected intervention for a newborn exposed to maternal Chlamydia would be to

a. use respiratory precautions
b. administration of a preventative vaccine
c. obtain a conjunctival scraping
d. use prophylactic antibiotics in labor
d. use prophylactic antibiotics in labor

During a newborn assessment, a NP notes hepatosplenomegaly, snuffles, and lymphadenopathy. The RN recognizes that these signs are consistent with

a. syphilis
b. gonorrhea
c. chlamydia
d. GBS infection
a. syphilis

An appropriate nursing intervention for a baby born to a mother who has hepatitis C virus is to

a. administer hepatitis B vaccine at birth
b. discourage breastfeeding
c. monitor for expected rash
d. cleanse the skin before vitamin K injection
d. cleanse the skin before vitamin K injection

A mother has a varicella rash 2 days before delivery. The RN should anticipate

a. administering VZIG (varicella zoster immunoglobulin) to the baby
b. performing routine newborn care
c. instituting only standard precautions
d. treating the baby with PCN
a. administering VZIG (varicella zoster immunoglobulin) to the baby

A baby delivered to a HIV positive mother is most likely to demonstrate

a. signs of thrush
b. RDS
c. no symptoms
d. chronic diarrhea
a. signs of thrush

A baby has IUGR, petechiae, hepatosplenomegaly, and microcephaly. CMV is suspected. The neonate should be observed for signs of

a. rash
b. hydrocephaly
c. cataracts
d. periventricular intracranial calcifications
d. periventricular intracranial calcifications

A common clinical sign of CMV is

a. hydrocephaly
b. LGA baby
c. hepatosplenomegaly
d. increased platelets
c. hepatosplenomegaly

A “blueberry muffin” rash is consistent with

a. rubella
b. herpes
c. hepatitis B
d. varicella
a. rubella

Contact transmission isolation is ordered on a baby. This type of isolation includes the mandatory use of

a. respiratory protective devices
b. mask
c. gowns
d. private room
c. gowns

A recommended routine measure to minimize infections in the newborn nursery is to

a. wash with soap and a brush for 15 minutes between newborn contacts
b. wear a long sleeve gown when holding the baby
c. scrub above the elbows for 10 minutes at the start of the nursing shift
d. discard flush solutions after each use
d. discard flush solutions after each use

Which parental statement following a neonatal discharge teaching session is accurate?

a. the baby needs a daily bath
b. lotion must be used for skin dryness
c. straining during bowel movements signals constipation
d. excessive suctioning can increase nasal drainage
d. excessive suctioning can increase nasal drainage

The RN is teaching new parents about neonatal care. The parents should notify the MD if the baby

a. has white patches that remain in the mouth after gently wiping
b. experiences wakeful periods in the early morning
c. sleeps around 16 hours a day
d. does not produce urine for 8 hours
a. has white patches that remain in the mouth after gently wiping

An appropriate safety measure for a baby is to

a. be certain the crib slat width is less than 2 and 3/8 inches
b. move the crib away from window blinds
c. change pacifiers weekly
d. use rear-facing car seats until the baby is 6 months old
a. be certain the crib slat width is less than 2 and 3/8 inches

A normal finding that indicates a baby is ready for discharge at 48 hours is

a. 20% weight loss
b. systolic blood pressure of 130
c. 3 stools since birth
d. HR greater than 200/min
c. 3 stools since birth

The parental situation that would raise greatest concern regarding newborn discharge is

a. questions about the hepatitis B vaccine
b. single, adolescent mother with no support system
c. maternal age of 35
d. maternal history of treated GBS
b. single, adolescent mother with no support system

A baby is discharged from the hospital before 48 hours. The baby should be evaluated within how many hours of discharge?

a. 11 hours
b. 24 hours
c. 48 hours
d. 96 hours
c. 48 hours

An MD directed nursing follow up clinic provides services for infants discharged at 24 hours or less. A newborn discharge at 12 hours should return to the hospital nursery

a. 1 day
b. 2 days
c. 3 days
d. 1 week
a. 1 day

What carries deoxygenated blood with fetal waste from fetus to placenta?

a. arterial duct
b. umbilical vein
c. oval opening
d. umbilical arteries
d. umbilical arteries

The connection between the pulmonary artery and the aorta, directs blood away from the lungs into systemic circulation

a. arterial duct
b. umbilical vein
c. venous duct
d. fetus
c. venous duct

The passage between the right and left atria of the heart, bypasses the lungs delivering most of the oxygenated blood directly to the body

a. oval opening
b. placenta
c. umbilical vein
d. fetus
c. umbilical vein

The organ interface between the embryo or fetus and the mother, separates bloodstreams, but allows the exchange of nutrients and waste products

a. placenta
b. oval opening
c. venous duct
d. fetus
a. placenta

Modified to allow the fetus to develop in utero, the fetus doesn’t require is own pulmonary circuit or lungs until birth

a. oval opening
b. umbilical vein
c. fetal circulation
d. fetus
c. fetal circulation

Receives blood from the umbilical vein and directs it to the inferior vena cava, acts as liver bypass

a. arterial duct
b. fetus
c. venous duct
d. placenta
c. venous duct

Carries oxygenated blood with maternal nutrients from placenta to fetus

a. umbilical vein
b. oval opening
c. umbilical arteries
d. arterial duct
a. umbilical vein

Stage in development from 8 weeks to birth

a. fetus
b. placenta
c. arterial duct
d. venous duct
a. fetus

A woman with a BMI of 18 gains 25 lbs during her pregnancy. This pregnancy is at increased risk for
a. cesarean delivery
b. Delivery of low birthweight delivery
c. preeclampsia
b. delivery of a low birthweight infant

During pregnancy, which of the following cardiovascular parameters increases by 60%
a. blood volume
b. plasma volume
c. red cell mass
b. plasma volume

When pregnancy, a woman with epilepsy has a higher risk of seizures
a. during labor through 24 hours postpartum
b. from 72 hours to 1 week pp
c. in the third trimester
a. during labor thru 24 hrs. pp

The risk of developing diabetes in women with a history of gestational diabetes is
a. increased only if there are also multiple diabetes risk factors
b. increased with onset within 10-15 years in about 50% of those women
c. the same as women who do not have gest. diabetes
b. increased with onset within 10-15 years in about 50% of those women

A woman recieves magnesium sulfate during labor for gestational hypertension. What effect can this have on newborn?
a. elevated cortisol levels
b.hypoglycemia
c.muscle weakness
c. muscle weakness

A pregnant woman was diagnosed with varicella-zoster at 32 weeks gestation. She is now in active labor at 40 weeks gestation. The nurse would expect that which of the following would be appropriate in order to ensure the health of the newborn.
a. admister zoster immunoglobulin to newborn at delivery
b. admister zoster immunoglobulin to patient before delivery
c. the mother’s antibodies will be protective so no action is needed
c. the mother’s antibodies will be protective so no action is needed

In order to decrease the risk of fetal death to that of a non-smoker, a woman must quit smoking
a. before the pregnancy occurs
b. by the end of the first trimester
c. by the end of the 2nd trimester
b. by the end of the first trimester

A patient in preterm labor at 30 weeks received indocin, the newborn should be observed for?
a. duodenal atresia
b. Hirschsprung disease
c. necrotizing entercolitis (NEC)
c. necrotizing enercolitis (NEC)

What are the characteristics of dizygotic pregnancy?
a. 1 placenta and 1 amniotic sac
b. 1 placenta and 2 amniotic sacs
c. 2 placentas and 2 amniotic sacs
c. 2 placentas and amniotic sacs

A woman is diagnosed with oligohydramnios during her pregnancy. the fetus/newborn should be evaluated for
a. diabetes insipidus
b. klippel-feil syndrome
c. polycystic kidney disease
c

A fetus at 36 weeks receives a BBP score of 6. the expectated management is
a. immediate delivery
b. repeat in 24 hrs
c. schedule the next test in one week
b

In evaluating the patient census for staff assignments, the pp nurse is told that a pateint in the high risk unit may be delivered and transferred to pp after delivery. A L/S ratio was performed with result of 2:0 the nurse would expect the following to occur?
a. A PG (phosphatidyglycerol) test needs to be performed
b. the lungs are mature, and delivery will occur
c. the lungs are still immature, and the L/S will be repeated in 1 week
b

Which of the following is a limitation of an alpha-fetaprotein (AFP) test?
a. closed neural tube defects are not detected
b. it will not detect down syndrome
c. its best sensitivity is at 20 weeks
a

A postpartum woman’s prenatal history shows a quad screen with a maternal serum (MSAFP) lower than normal and higher than HCG and Inhibin A. The nurse can expect the diagnosis of the newborn?
a. neural tube defect
b. Trisomy 21
c. Underestimated gestational age
b

An umbilical gas ( artery) reveals the following ph: 7.27, PC02: 50mm Hg; HC03: 23 meq/Ll base excessl -3.6. this represents
a. normal value
b. metabolic acidosis
c. respiratory acidosis
a

Umbical blood gas values are used to?
a. determine etiology of any immediate complication
b. direct neonatal resuscitation efforts
c. establish the metabolic status of the fetus
c

Ultrasound for antepartum testing uses high frequency sound waves to produce an image that varies based on the
a. amount of fetus moves
b. density of the structure under the transducer
c. strength of the reflected energy
b

In evaluating a cord blood gas, if the pH is low, what other blood gas parameter is used to determine if the acidosis is respiratory or metabolic?
a. PC02
b. HCO3
c. Po2
a

Breech presentation increases risk for
a. cord prolapse
b. fetal clavicle fracture
c. shoulder dystocia
a

A newly delivered postpartum patient received received terb and corticosteroids during labor. A potential drug interaction that the pp nurse should watch for in the woman is
a. hyperglycemia
b. ketoacidosis
c. pulmonary edema
a

A postpartum patient has a spinal headache. The nurse knows that this can occur as a consequence of epidural due to
a. injured nerve pathways
b. puncture of the dura
c. too rapid adminstration
b

Shoulder dystocia places the pp woman at increased risk for
a. endometritis
b. hemorrhage
c. urinary retention
b

A newborn is at a higher risk for jaundice when delivered by
a. cesarean section
b.forceps
c. vacuum
c

A woman who is 4 hr pp asks the nurse how much weight she might have lost with the deliver. The nurse would know to tell her the average wgt loss at birth is
a. 10 lbs
b. 12 lbs
c. 14 lbs
b

When assessing a woman who is 1 day pp, the nurse would expect her fundus to be palpated
a. at umbilicus
b. 1cm above umbilicus
c. 1cm below umbilicus
c

Within the first 2 days after birth, progesterone levels fall causing what acid base change?
a. Increased PaCO2
b. Decreased PCO2
c. Decreased PaO2
a

Compared to the value seen in the 3rd trimester, the catabolic process of involution causes an increase in which of the following values pp?
a.BUN
b. creatinine clearance
c. proteinuria
a

A pp patient on her 2nd day is concerned that she has been urinating large amounts and frequently. Her output is 3000ml, the appropriate response by the nurse
a.contact her care provider for an antibiotic order
b. explain that this is a normal physilogic response to delivery
c. obtain a clean catch urine
b

Forceps delivery can result in what complication in the newborn?
a.blood clots
b.craniosynotosis
c.nerve injuries
c

To avoid a precipitous drop in intra-abdominal pressure during catheterization of a postpartum patint’s bladder, what is the maximum amount of urine that should be removed at one time?
a.700 ml
b. 800ml
c. 900 ml
b

A woman is 12 hours pp and has a wbc of 25,000 this indicates
a.an expected range
b.dehydration and a need to force
c.infection and a need for an antibiotic order
a

During pp, the low levels of placental lactogen, estrogen, cortisol, growth hormone, and insulinase result in
a.diuresis
b.increased angiotensin II
c. lower glucose levels
c

2 hours after delivery an examination of the uterine fundus finds it deviated from midline. The nurse should have the woman
a. flex her legs and reassess
b.further evaluated for infection
c. void to empty the bladder
c

When assessing a new pp patient, the nurse would expect her blood pressure to be
a. consistent w baseline blood pressure during pregnancy
b. increased from the work of labor
c.lower than normal
a

Following delivery, an insulin dependent diabetic will have her insulin dose reduced because.
a. her metabolism is in a state of flux immediately pp
b. nutritional intake is still low and activity
c. the anti-insulin effect ceases after placental separation
c

A woman who is 12 hours pp and has a history of asthma, is noted to have uterine atony. the patient’s v/s are bp 150/90, pulse 100 and resp 14. her temp is 99.4. the nurse would expect the medication appropriate
a. hemabate
b. methergine
c. oxytocin
c

scopalamine is classified as a category 6 drug by the American academy of pediatrics. this means that the drug
a. can be used during breastfeeding but watch for signs of poor feeding
b. is compatible w breastfeeding
c. should not be used while breastfeeding
b

A nurse discovers that an order for rh immune globulin (RhoGam) to be given 72 hrs past delivery was missed. It has been 4 days and the pt is to be d/c home. Follwing standard medication error, what is the correct action regarding the administration of the RhoGam at this time?
a. a Kleihauer-Betke smear should be obtained before adminstration of Rhogam
b. Administer 300 mg of Rhogam
c. Adminster 300 mg of Rhogam now and check Kleihauer-Betke at 6 weeks
b

A pp woman is given RH immune globin (Rhogam) at two days pp. the nurse would alert the woman to what possible side effects.
a. increased lochia for a few hours following administration
b. soreness at the site of injection site
c.given at the same time.
b

a pp woman has an order for Rh immune globulin and rubella vaccine the nurse knows that the rubella vaccine should be.
a. adminstered at the pp office visit
b. delayed until 24 hrs after the Rh immune globulin is administered
c. given at same time
c

The unsensitized Rh-negative mother whose baby is Rh positive should
a. have the father’s blood type determined
b. receive 300 mcg of Rho (D) by 72 hrs of birth
c. require no further treatment
b

after pains are due to
a. hormonal shifts that affect the muscle
b. physiologic stress from the birth process
c. uterine contractions during involution
c

A pp woman who is not breastfeeding states that her mother recommends she bind her breast to prevent painful engorgement. The nurse would advise the patient that binding
a. and an order for a diuretic will shorten the engorgement
b. and restriction of fluids for 2 days is recommended
c.may lead to more leaking
c

The American Academy of Pediatrics recommend 400 IU of vitamin D supplement for all infants beginning at what age?
a. 2 months
b. 4 months
c. soon after birth
c.

The lactation initiation stage is dependent on
a. a cascade of hormonal changes
b. early nursing or milk removal
c. prolactin
a

A new breastfeeding mother wants to begin a reduction diet to lose weight. The min number of kcal a lactating woman should consume per day is
a. 1800
b.2000
c.2200
a

While in the hospital the mother of a breastfeeding term newborn is observed to have flat nipples and is having difficulty getting baby to latch on properly. The nurse should recommend
a. nipple rolling
b. nursing w a nipple shield
c. wearing breast shells btw feedings
c.

a pt reports painful nipples. the nurse knows that
a. a lack of nipple preparation can result in pain
b. nursing too long can cause pain
c. specific areas of pain can have a specific cause
c

a pp mother who would like to breastfeed her baby is diagnosed w hepatitis A virus before delivery. what is the recommendation regarding breastfeeding for this patient?
a. Administer immunoglobin and hepatitis antivirus vaccine to the newborn and allow nursing
b. do not nurse due to the risk of horizontal transmission from the mother
c. pump and discard milk for 1 week following vaccine administration to the newborn
a

A pp mother diagnosed with cytomegalovirus (CMV) during 3rd trimester delivers a baby at 40 weeks. what is the recommendation regarding breastfeeding?
a. breastfeed after deilivery
b. breastfeed after two or more weeks of treatment of the mother
c. restrict breastfeeding until treatment is completed
a

When deciding on a mechanical breast pump, which of the following features is the most important?
a. cycle pressure sustained for 2 seconds or a rate of 30 pumps per min
b. manually controlled pressure
c. negative pressure of >250mm/hg to initiate milk flow
a

The cause of sudden late onset of pp hemorrhage secondary to retained placental fragments is due to
a. increased uterine contraction activity
b.nectrotic tissue separation from the uterus
b. weakness in the uterine musculature
ab

Clinical signs of hemorrhage such as b/p 90/70 pule 118, tachypnea can be seen when the blood loss reaches
a. 900
b. 1200
c. 1800
b

THe most common pp infection is
a. endometritis
b. mastitis
c.UTI
a

A pp patient had documented prenatal exposure to parovovirus. a serologic test shows pre-existing IgG antibodies the patient is concerned about the infection. Her hx indicates that which of the following recommendations would be appropriate?
a. breastfeeding may expose the newborn
b. recurrence is unlikely
c. she should have the test repeated in 3 weeks
b

A woman 4 hours pp reports sudden, excrutiating vulvar pain and severe rectal pressure. the patient is likely exhibiting symptoms of
a. Bartholin gland cyst
b. vulvar edema
c. vulvar hematoma
c

Following a cesarean birth, the insulin infusion of a diabetic woman should be
a. decreased by 50% until eating is resumed
b. maintained at her pre-delivery level the first 24 hours
c. stopped and sq insulin restarted
a

A pt is admitted to pp unit w a hx of pre-eclampsia during labor. what percent of eclampsia cases will occur during the pp period.
a. 10%
b. 20%
c. 30%
c

A pt has a 2 hr glucose tolerance (GTT)result of 140. the nurse would expect which of the following recommendations for the pt to obtain the best assessment of her current risk of diabetes?
a. annual fasting blood sugars
b. two hr GTT at 6 wks and every 3 yrs if normal
c. two hr GTT at 6 weeks and then annually if normal
b

on pp day 2 , a woman exhibits signs of sleep disturbances, irritabiltiy, agitation, and confusion.. the nurse would suspect
a. pp psychosis
b. ptsd
c. sepsis
a

The process that triggers the metabolism of brown fat in the neonate occurs for what purpose?
a. nutritional supplement
b. o2 stimulation when levels fall below 90
c. thermoregulation
c

At birth the most potent stimulus for constriction of the ductus arteriosis is an increase in
a. Co2
b. Pao2
c. PC02
b

A normal breathing pattern for a two hr old neonate may include
a. brief periods of apnea
b. seesaw breathing
c. tachypnea
a

At what rate per minute should chest compressions be done
a. 80
b. 90
c. 100
b

What size endotracheal tube should be available to resuscitate a 36 wk 2500 gm newborn
a. 2.5mm
b.3.0 mm
c. 3.5mm
c

The use of heat shield will prevent what type of heat loss
a. conduction
b. convection
c. radiation
c

When considering thermoregulation in the nbn, the loss of heat to moving air at the skin surface is called
a. conduction
b. convection
c. radiation
b

A nbn with hypothermia should be observed for
a. dehydration
b. metabolic acidosis
c. tachypnea
b

A critical finding of hyperthermia is
a.hypoglycemia
b. shivering
c. tachycardia
c

Which of the following temp of a term newborn is considered normal
a. 36. C
b. 37 C
c. 38 C
b.

Gavage feedings are frequently needed to meet the nutritional needs of preterm infants bc
a. lactose enzyme activity is not adequate
b. suck, swallow, and breathing reflexes are uncoordinated
c. hyperbilirubinemia is likely
b

Surfactant production for an infant with RDS is inhibited b/c of
a. prematurity
b. infection
c. hypoglycemia
a

Which statement is not true about RDS
a. It is characterized by atelectasis
b. it may be induced by hypothermia in a preterm infant
c. with adequate supportive care, it is self-resolving in approx 72 hrs.
c

The best indicator an infants need for oxygen is
a. resp rate
b. skin color
c. arterial Pa02
c

What problem may the LGA infant experience
a. PDA
b. facial nerve damage
c. poor suck swallow, breathing coordination
b

Why is hyperbilirubinemia of special concern in preterm infants.
a. immature liver function
b. poor vascular system
c. immature endocrine function
a

To assess the fontanelles, the newborn should be in what position
a. lying flat on its back
b. sitting position
c. turned to one side
b

The nurse notes hair on the ears of a newborn. she knows that this is seen in babies born to mothers who are
a. diabetic
b. drug addicted
c. post dates
a

When providing cord care to a newborn, which of the following should be used.
a. alcohol
b. betadine
c. sterile water
a

A neonate born to a mother whose HBsAg status is unknown should have
a. hepatitis B vaccine and HBIG adminstered only after the mother
b. the first dose of hep B within the first 12 hrs after birth
c. the same schedule as a neonate born to HBsAG neg mother
b

The pp discharge instructions given to an HBsAg mother whose newborn received Hepatitis B vaccine at birth should include the follow up vaccines to be administered to the newborn at what ages?
a. 1 month and 5 mos
b. 2 mos and 4 mos
c. 3 mos and 6 mos
b

When the nurse examines the umbilical cord of a newborn, she notes an umbilical cord hernia, This is more commonly seen in which ethnic population
a. African American
b. Caucasian
c. Latino
a.

On the day of birth, a vitamin K injection is necessary bc of which of the following physiological states present in the first few days of life
a. absence of normal flora in the gut for synthesis of vit K
b. coagulation factors are unstable
c.the newborn liver does not have complete physiological function
a

What class of antibiotic can interfere with the binding of bilirubin to protein in the first month?
a. Aminoglycosides
b. penicillins
c. sulfas
c

A newborn observed to have hepatomegaly, cardiac gallop, tachypnea, tachycardia, cyanosis, diaphoresis, and feeding problems most likely has
a. CHF
b.tetralogy of fallot
c. ventricular septal defect
b

A few hours after administering the eye prophylaxis to the newborn, the nurse notes edema and a greenish yellow discharge. This is most likely a reaction to
a. erythromycin
b silver nitrate
c. tetracycline
b

A newborn is noted to have cyanosis when quiet but is pink when crying. this is most likely due to
a. choanal atresia
b. pneumothorax
c. tetrology of fallot
a

Which of the following newborn diagnoses is a result of delayed re-absorption of fetal lung fluid production.
a. pneumothorax
b. respiratory distress syndrome
c. transient tachypnea of the newborn
b

Newborns delivered by what method are more likely to develop ttn of the newborn
a. c/s
b. induction
c. vacuum
a

a newborn is delivered w meconium staining who is responsive should be managed how
a. always suction a the perineum
b. no suction if baby is active and does not require resuscitation
c. suction only after the airway has been inspected
b

at birth the newborn is noted to have absent breath sounds on the left with heart sounds shifted to the right. the abd is noted to be scaphoid and ventilation is difficult. the most likely diagnosis is
a. congential pneumothorax
b. diaphragmatic hernia
c. transposition of the great vessels
b

Which of the following is chacteristic of jitterness and not of seizures?
a. eye deviations
b. motion can elicit
c.tachycardia
b

Phototherapy via a fiber optic blanket is ordered for a newborn who is 30 hours old. The mother should be instructed to do which of the following?
a. apply lotion to the skin during tx to prevent drying
b. increase fluids intake to 25% above daily normal
c. no change in care is required
c

In a 12 hour newborn, the nurse notes a tremor in which the amplitude and rate of the alternating mvmt is equal. this is characteristic of
a. cold stess
b jitteriness
c. seizure
c

A woman delivers a newborn with spina bifida. She questions what her chances is that another baby would have a similar problem. The nurse knows that the risk of another pregnancy with a neural tube defect is
a. a random event
b. ten times more likely the next pregnancy
c. the same as the first
b

A neonate has scrotal swelling which is soft, non-tender and transilluminates there are no palpable masses. the most likely diagnosis is
a. cyrptorchidism
b. hydrocele
c. hypospadias
b

Group B streptococcus (GBS) and what other organism are currently the most common pathogens seen with neonatal sepsis?
a. chlamydia
b. E. coli
c. Klebsiella
b

Born to a mother of HIV, which of the following tests would be appropriate for the nb.
a. ELISA testing at birth w cord blood
b. DNA and polymerase chain reaction (PCR) test before 48 hours of age
c. western blot test at birth and repeat at discharge
b.

The specific tests that are mandated in newborn screening for inborn errors of metabolism are determined by
a. hospital policy
b. individual state law
c. the CDC
b

A newborn whose mother was a poorly controlled diabetic during the third trimester is at risk for
a. hypercalcemia
b. magnesium deficiency
c. polycythemia
c

Leave a Comment

Scroll to Top